SSC CGL Tier-I 04 September 2016 Shift-I Previous Year Paper

SSC CGL Tier 1 2016 4 Sep Shift 1

Q. 1 Select the related word from the given alternatives :

Play : Actor :: Concert :

A. Symphony

B. Musician

C. Piano

D. Percussion

 

Q. 2 Select the related letters from the given alternatives:

EGIK : FILO :: FHJL : ?

A. JGMP

B. JGPM

C. GJMP

D. GMJP

 

Q. 3 Select the related number from the given alternatives :

583 : 295 :: 486 : ?

A. 291

B. 378

C. 487

D. 581

 

Q. 4 Find the odd word from the given alternatives :

A. Fire

B. Light

C. Gas

D. Water

 

Q. 5 Find the odd word from the given alternatives:

A. ECBY

B. RTUX

C. GEDA

D. WUTQ

 

Q. 6 Find the odd number pair from the given alternatives.

A. 52-61

B. 43-54

C. 72-83

D. 18-29

 

Q. 7 Arrange the following words as per order in the dictionary.

1. Fraudulent

2. Fraught

3. Fraternity

4. Franchise

5. Frantic

A. 5, 4, 1, 2, 3

B. 4, 5, 3, 1, 2

C. 4, 3, 5, 1, 2

D. 3, 4, 5, 2, 1

 

Q. 8 Which one set of letters when sequentially placed at the gaps in the given letter series shall complete it?

H __ JH __ IJHHI __ HH __ JH

A. I H J I

B. H I H I

C. I H I J

D. H J H J

 

Q. 9 Which number will complete the series?

63, 72, 81, 90, ___, 108

A. 80

B. 99

C. 100

D. 117

 

Q. 10 Rajiv is the brother of Arun. Sonia is the sister of Sunil. Arun is the son of Sonia. How is Rajiv related to Sunil?

A. nephew

B. son

C. brother

D. father

 

Q. 11 A mother is five times older than her daughter. After 5 years, she would be 3 times older than her daughter. Find the mother’s present age.

A. 20 years

B. 22 years

C. 25 years

D. 29 years

 

Q. 12 From the given alternative words, select the word which cannot be formed using the letters of the given word:

PRONOUNCEMENT

A. MOUNT

B. CEMENT

C. PAVEMENT

D. NOUN

 

Q. 13 If SYSTEM is written as RXRSDL. How can CORRECT be written in that code?

A. BNQQDBS

B. BQQNDBS

C. BNQQBDS

D. BNQDQBS

 

Q. 14 If A means +, B means x, C means ÷, D means -, then the value of given equation will be 9A2B6D4C2

A. 16

B. 19

C. 27

D. 30

 

Q. 15 If 4 x 3 = 14; 5 x 4 = 18; 6 x 5 = 22, then find the value of 7 x 6.

A. 20

B. 26

C. 30

D. 42

 

Q. 16 Select the missing number from the given responses:

A. 6

B. 9

C. 5

D. 4

 

Q. 17 Rita travelled 35 km from a point towards South and then turned left and travelled 20 km and finally turned left again and travelled 35 km. In which direction is she from the starting point?

A. East

B. West

C. North

D. South

 

Q. 18 Consider the given statement/s to be true and decide which of the given conclusions/assumptions can definitely be drawn from the given statement. Statement: To achieve economic development, people should work hard.

Conclusions:

I. Economic development is directly related to people’s hard work.

II. Working hard by all the people is impossible.

A. Only I follows

B. Only II follows

C. Both I and II follow

D. Neither I nor II follows

 

Q. 19 Find the number of triangles in the figure (1).

A. 12

B. 18

C. 22

D. 26

 

Q. 20 Which of the answer figures (1), (2), (3), (4) best represents the relation between: Shirt, Garments, Clothes

A. (1)

B. (2)

C. (3)

D. (4)

 

Q. 21  Which answer figure among (1), (2), (3), (4), will complete the pattern in the question figure (A)?

A. (1)

B. (2)

C. (3)

D. (4)

 

Q. 22 From the given answer figures (1), (2), (3), (4), select the one in which the question figure (A) is hidden/embedded.

A. (1)

B. (2)

C. (3)

D. (4)

 

Q. 23 A piece of paper is folded and punched as shown in figure (A). From the given answer figures, indicate how it will appear when opened (among figure (1), (2), (3), (4))

A. (1)

B. (2)

C. (3)

D. (4)

 

Q. 24 The mirror image of a word is seen as given in figure (A). What is the actual word among (1), (2), (3), (4)?

A. (1)

B. (2)

C. (3)

D. (4)

 

Q. 25 In the question, a word is represented by only one set of numbers as given in any one of the alternatives. The set of numbers given in the alternatives are represented by two classes of alphabets as in two matrices given in figure (1). The columns and rows of Matrix I are numbered from 0 to 4 and that of Matrix II are numbered from 5 to 9. A letter from these matrices can be represented first by its row and next by its column, e.g., B can be represented by 00, 13, etc, and A can be represented by 55, 69, etc. You have to identify the set for the word ‘LION’.

A. 03, 55, 76, 33

B. 11, 65, 77, 22

C. 23, 79, 85, 43

D. 11, 88, 99, 22

 

Q. 26 Besides CO2, the other greenhouse gas is?

A. CH4

B. N2 N2

C. Ar Ar

D. O2

 

Q. 27 National Income of India is compiled by

A. Finance Commission

B. Indian Statistical Institute

C. National Development Council

D. Central Statistical Organisation

 

Q. 28 The power of the Supreme Court of India to decide disputes between the Centre and the States falls under its

A. Advisory jurisdiction

B. Original jurisdiction

C. Appellate jurisdiction

D. Jurisprudence

 

Q. 29 The word ‘Satyamev Jayate’ have been derived from which Upanishad?

A. Akshi Upanishad

B. Mundaka Upanishad

C. Garuda Upanishad

D. Mahavakya Upanishad

 

Q. 30 When Granite rocks get metamorphosed, they form

A. Quartzite

B. Gneisa

C. Marble

D. Slate

 

Q. 31 Which of the following is a commercial source of energy?

A. Agricultural waste

B. Dried dung

C. Sun

D. Natural gas

 

Q. 32 Backward bending supply curve belongs to which market?

A. Capital

B. Labour

C. Money

D. Inventories

 

Q. 33 In IT, associative memory is called as

A. Virtual memory

B. Cache memory

C. Content addressable memory

D. Main memory

 

Q. 34 Addition of suitable impurities into semiconductor, is called

A. Doping

B. Mixing

C. Forming

D. Diluting

 

Q. 35 Kyoto Protocol is associated with:

A. Species conservation

B. Climate change

C. Wetland conservation

D. Medicinal plants

 

Q. 36 Ebrahim Alkazi is an eminent personality in which one of the following area?

A. Heart surgery

B. Civil aviation

C. Theatre

D. Painting

 

Q. 37 Skoda, which launched sedan ‘Rapid’, is an automobile manufacturer based in

A. Japan

B. France

C. South Korea

D. Czech Republic

 

Q. 38 Sattriya is a classical dance form of ______________

A. Manipur

B. Uttar Pradesh

C. Assam

D. Andhra Pradesh

 

Q. 39 P. V. Sindhu defeated Wang Yihan to enter the semi final of womens’ singles of badminton at the Rio Olympics. Wang Yihan belongs to _________________

A. Indonesia

B. Taiwan

C. China

D. Japan

 

Q. 40 If a budget is defeated in the legislature of a state then

A. The Finance Minister alone has to resign

B. The Finance Minister concerned has to be suspended

C. The council of Ministers along with the Chief Minister has to resign

D. Reelection have to be ordered

 

Q. 41 Which one of the following papers was edited by Gandhiji in South Africa?

A. Indian Opinion

B. Harijan

C. Young India

D. Indian Mirror

 

Q. 42 In which of the following cities India’s first water metro project was recently launched?

A. Kolkata

B. Kochi

C. Visakhapatnam

D. Mumbai

 

Q. 43 Meibomian glands are located in

A. Eye

B. Ear

C. Nose

D. Skin

 

Q. 44 In which city is India’s first Post Office ATM opened?

A. Chennai

B. New Delhi

C. Hyderabad

D. Mumbai

 

Q. 45 Arsenic problem in India is primarily due to

A. Overexploitation of coal in Bihar and Bengal

B. Overexploitation of arsenopyrite in the hinterland

C. Overexploitation of groundwater in the affected areas

D. Overexploitation of surface water in the affected

 

Q. 46 Men’s Singles Wimbledon Championship 2016, is bagged by

A. Andy Murray

B. Milos Ravnic

C. Roger Federer

D. Novak Dijokovic

 

Q. 47 Which of the following vitamins are water soluble?

A. Vit. A and Vit. B

B. Vit. B and Vit. C

C. Vit. C and Vit. D

D. Vit. A and Vit. K

 

Q. 48 In terms of economics, if it is possible to make someone better off without making someone worse off, then the situation is

A. Insufficient

B. Efficient

C. Optimal

D. Paretosuperior

 

Q. 49 Greenhouse Effect means

A. Pollution in houses in tropical region

B. Prevention of ultraviolet radiation by the ozone layer

C. Trapping of solar energy due to atmospheric gases

D. Damage to green painted buildings

 

Q. 50 General Dyer, who was responsible for Jallianwala Bagh massacre, was shot dead by?

A. Hasrat Mohani

B. Vir Savarkar

C. Udham Singh

D. Jatin Das

 

Q. 51 X can do a piece of work in ‘p’ days and Y can do the same work in ‘q’ days. Then the number of days in which X and Y can together do that work is

A. (p + q)/2

B. 1/p + 1/q

C. pq/(p + q)

D. pq

 

Q. 52 A shopkeeper marks his goods 40% above the cost price and allows a discount of 25% on it. His gain per cent is

A. 5%

B. 10%

C. 15%

D. 20%

 

Q. 53 The ratio of the present ages of two boys is 3 : 4. After 3 years, the ratio of their ages is equal to will be 4 : 5. The ratio of their ages after 21 years will be

A. 14 : 17

B. 17 : 19

C. 11 : 12

D. 10 : 11

 

Q. 54 The cost price of 25 books is equal to the selling price of 20 books. The profit percent is:

A. 20%

B. 22%

C. 24%

D. 25%

 

Q. 55 One number is 25% of another number. The larger number is 12 more than the smaller. The larger number is:

A. 48

B. 16

C. 4

D. 12

Q. 56 A train, 500 metre long, running at a uniform speed, passes a station in 35 seconds. If the length of the platform is 221 metre, the speed of the train in km/hr is

A. 72 1/35

B. 74.16

C. 24.76

D. 78.54

 

Q. 57 If the simple interest on Rs. 400 for 10 years is Rs. 280, the rate of interest per annum is :

A. 7%

B. 7 1/2%

C. 7 1/4%

D. 8 1/4%

 

Q. 58 If a + b = 2c, then the value of a/(a – c) + c/(b – c) is equal to (where a ≠ b ≠ c)

A. -1

B. 1

C. 0

D. 1/2

 

Q. 59 If x + 1/x = 5, then the value of x/(1 + x + x^2) is

A. 1/5

B. 1/6

C. 5

D. 6

 

Q. 60 G and AD are respectively the centroid and median of the triangle ΔABC. The ratio AG : AD is equal to

A. 3 : 2

B. 2 : 3

C. 2 : 1

D. 1 : 2

 

Q. 61 A point P lying inside a triangle is equidistant from the vertices of the triangle. Then the triangle has P as its

A. Centroid

B. Incentre

C. Orthocentre

D. Circumcentre

 

Q. 62 If sinθ + cosθ = 1, then sinθ . cosθ is equal to

A. 0

B. 1

C. 1/2

D. -1/2

 

Q. 63 If 7 times the seventh term of an Arithmetic Progression (AP) is equal to 11 times its eleventh term, then the 18th term of the AP will be

A. 1

B. 0

C. 2

D. -1

 

Q. 64 The average age of eight teachers in a school is 40 years. A teacher among them died at the age of 55 years whereas another teacher whose age was 39 years joins them. The average age of the teachers in the school now is (in years)

A. 35

B. 36

C. 38

D. 39

 

Q. 65 If a^2/(b + c) = b^2/(c + a) = c^2/(a + b) = 1 then find the value of 2/(1 + a) + 2/(1 + b) + 2/(1 + c)

A. 0

B. 1

C. 2

D. 3

 

Q. 66 If 2x + 2/x = 3, then the value of x^3 + 1/x^3 + 2 is

A. 3/4

B. 4/5

C. 5/8

D. 7/8

 

Q. 67 There are two equal circles of radius 3cm each and distance between their centres is 10 cm. The length of one of their transverse common tangents is?

A. 7 cm

B. 9 cm

C. 10 cm

D. 8 cm

 

Q. 68 In ΔABC if the median 1/2 AD = BC, then ∠BAC is equal to

A. 90°

B. 45°

C. 60°

D. 75°

 

Q. 69 IF (sinθ + cosθ)/(sinθ – cosθ) = 3 then the value of sin^4θ – cos^θ is

A. 4/3

B. 3/4

C. 5/3

D. 3/5

 

Q. 70 A sphere has the same curved surface area as a cone of vertical height 40 cm and radius 30 cm. The radius of the sphere is

A. 5√5 cm

B. 5√3 cm

C. 5√15 cm

D. 5√10 cm

 

Q. 71 The angle of elevation of the top of a tower from a point A on the ground is 30°. On moving a distance of 20 metres towards the foot of the tower to a point B, the angle of elevation increases to 60°. The height of the tower in metres is

A. √3

B. 5√3

C. 10√3

D. 20√3

 

Question :- 72 – 75

Given is a pie chart in figure (1) showing the cost of gold in 2010, 2011, 2012 and

2013.

 

Q. 72 If the price of gold in 2013 is Rs. 31,500 per 10 gram, then the price of gold in 2011 per 10 gram is

A. Rs 17000

B. Rs 17500

C. Rs 18000

D. Rs 18500

 

Q. 73 The ratio of the price of gold in the two years 2010 and 2013 is

A. 1 : 2

B. 1 : 3

C. 1 : 4

D. 1 : 5

 

Q. 74 The percentage increase in the price of gold from the year 2011 to 2013 is

A. 50%

B. 60%

C. 70%

D. 80%

 

Q. 75 The ratio of percentage increases in price of gold from 2011 and 2012 to 2013 is

A. 6 : 5

B. 7 : 5

C. 8 : 5

D. 9 : 5

 

Question:_ 76-79

In the following question, out of the four alternatives, choose the word which best expresses the meaning of the given word.

 

Q. 76 EXAGGERATE

A. Magnify

B. Imagine

C. Reinforce

D. Reiterate

 

Q. 77 GREGARIOUS

A. Unsociable

B. Unsympathetic

C. Ungrateful

D. Unattractive

 

Q. 78 Four words are given, out of which only one word is spelt correctly. Choose the correctly spelt word.

A. Serendipty

B. Serendipity

C. Serndipty

D. Sernidipity

 

Question:-79-81

In the following questions, one part of the sentence may have an error. Find out which part of the sentence has an error and choose the alternative. If the sentence is free from error, choose the “No error” option.

 

Q. 79 Considering the (1)/ gravity of the problems (2)/ an early reply has expected. (3)/ No error (4)

A. (1)

B. (2)

C. (3)

D. (4)

 

Q. 80 The Statesman has the (1)/ larger circulation (2)/ of all English dailies. (3)/ No error (4)

A. (1)

B. (2)

C. (3)

D. (4)

 

Q. 81 I am very anxious (1)/ to know how are you (2)/ and mother are doing (3)/ No error(4)

A. (1)

B. (2)

C. (3)

D. (4)

 

Question:-82-84

In the following questions, the sentences given with blanks are to be filled with an appropriate word(s). Four alternatives are suggested for each question. 

Q. 82 Everything ___________________ carefully checked.

A. has been

B. have been

C. are being

D. is been

 

Q. 83 I’ve __________________ finished painting the house.

A. about

B. nearing

C. towards

D. almost

 

Q. 84 She _________________________ made him angry.

A. have absolutely

B. has certainly

C. have certainly

D. has absolutely

 

Question:-85-90

In each of the following questions, four alternatives are given for the Idiom/Phrase. Choose the alternative which best expresses the meaning of the Idiom/Phrase.

 

Q. 85 Burn your boats

A. Have a burning desire to win

B. Become extremely tired after working very hard

C. Do something that makes it impossible to return to the previous situation

D. Want to spend money as soon as you get it

 

Q. 86 Dressing down

A. Apply bandage

B. Wear an expensive gown

C. Give a scolding

D. Pretend

 

Q. 87 Null and void

A. Invalid

B. Observant

C. Uncontrolled

D. Homeless

 

Question:-88-90

In the following questions, out of the four alternatives, choose the one which can be substituted for the given words/sentences 

 

Q. 88 Write or carve words on stone or paper

A. Sketch

B. Imprint

C. Affix

D. Inscribe

 

Q. 89 Unable to pay one’s debt

A. Insolvent

B. Impute

C. Indebt

D. Obligate

 

Q. 90 Trouble and annoy continually

A. Complaint

B. Harass

C. Punish

D. Oppress

 

Question:-91-95

In each of the following questions, a sentence/ a part of the sentence is quoted. Four alternatives are given to the quoted part which will improve the sentence. Choose the correct alternatives corresponding to it.

 

Q. 91 The Chairman’s harsh words “like” insult to injury to the embarrassed speaker

A. added

B. evoked

C. invoked

D. No improvement

 

Q. 92 He is “in” of sorts today.

A. out

B. on

C. outside

D. no improvement

 

Q. 93 She “told to me” the news

A. said me

B. said to me

C. told me

D. No improvement

 

Q. 94 The philanthropist “bestowned” a lakh to the orphanage.

A. granted

B. parted with

C. donated

D. No improvement

 

Q. 95 “There are a better ways of solving the problem”

A. There are different ways of solving a problem

B. There is a better way of looking at the problem

C. There are better ways of solving the problem.

D. No improvement.

 

Question:- 96-100

In general it is better to use too little make-up than too much. The audience should not be aware that the actor’s face is painted. For the actor who is playing his own age, the artist uses makeup to strengthen the features, particularly eyes and mouth, and to add lifelike colour to the face. Character makeup does these things in addition to transforming the face to another age, another type or another race. This transformation, particular for young actors playing old characters, can be helped greatly by hats and hairdos. Make-up consists of applying a base colour, then modelling the face by highlighting and shadowing (sinking the cheeks, for example, with a darker colour). Sometimes, modelling is done by applying false (putty or plastic) noses, enlarged eyebrows, or scars. Lines to suggest wrinkles are drawn on with a dark makeup pencil (brown or maroon, not black) or brush. Each line is highlighted with another line, either white or light tint of the base colour. Lips are outlined and coloured, and a similar colour is applied to the cheeks. After make-up is complete, powder is applied. 

 

Q. 96 The artist uses makeup to strengthen the features, particularly eyes and mouth, and to add life like colours to the face for the actor who is playing ____________

A. the lead role

B. the old man or woman

C. his own age

D. the role of the clown

 

Q. 97 What do artists use to help young actors playing old men?

A. Wig and moustache

B. Hats and hairdos

C. Wheelchairs and walking sticks

D. False noses, enlarged eyebrows or scars

 

Q. 98 What is the correct sequence for make-up?

A. Modelling the face by highlighting and shadowing then applying a base colour

B. Highlighting and shadowing, then applying a base colour and modelling the face

C. Applying a base colour, then modelling the face by highlighting and shadowing

D. Shadowing and modelling, then applying a base colour for highlighting

 

Q. 99 Lines are drawn with a dark make-up pencil or brush to suggest ______________

A. Dimples

B. Wrinkles

C. Smiles

D. Pimples 

 

Q. 100 When is powder usually applied? 

A. Before the make-up

B. As make-up foundation

C. After the make-up

D. During the make-up

 

 

Answer Sheet 
Question 1 2 3 4 5 6 7 8 9 10
Answer B C B D B A B A B A
Question 11 12 13 14 15 16 17 18 19 20
Answer C C A B B C A A B A
Question 21 22 23 24 25 26 27 28 29 30
Answer D A C D D A D B B B
Question 31 32 33 34 35 36 37 38 39 40
Answer D B C A B C D C C C
Question 41 42 43 44 45 46 47 48 49 50
Answer A B A A C A B A C C
Question 51 52 53 54 55 56 57 58 59 60
Answer C A D D B B A B B B
Question 61 62 63 64 65 66 67 68 69 70
Answer D A B C C D D A D C
Question 71 72 73 74 75 76 77 78 79 80
Answer C B B D B A A B C B
Question 81 82 83 84 85 86 87 88 89 90
Answer B A D B C C A D A B
Question 91 92 93 94 95 96 97 98 99 100
Answer A A C C C C B C B C

SSC CGL Tier-I 03 September 2016 Shift-I Previous Year Paper

SSC CGL Tier 1 2016 3 Sep Shift 1

Q. 1 Select the related word from the given alternatives :

Owl : Hoots : : Hen : ?

A. Chirps

B. Clucks

C. Coos

D. Cackles

 

Q. 2 Select the related letters from the given alternatives.

AKU:?: :·CMW: DN

A. BGL

B. BLQ

C. BGQ

D. BLV

 

Q. 3 Select the related number from the given alternatives.

5:100::7:?

A. 49

B. 196

C. 91

D. 135

Q. 4 Find the odd word from the · given alternatives

A. time

B. skill

C. interest

D. knowledge

 

Q. 5 Find the odd letters from the given alternatives

A. CPA

B. REB

C. QUD

D. AOT

 

Q. 6 Find the ocld number pair from the given alternatives

A. 15-21

B. 32-41

C. 22-27

D. 31-35

 

Q. 7 Arrange,the following words as per order in the dictionary and then choose the one which comes last.

A. Qualify

B. Quarter

C. Quarrel

D. Quaver

 

Q. 8 A series is given, with one term missing._Ghoose the correct alternative from_ the given ones that will complete the series.

DF, GJ,.KM, NQ, RT,?

A. EI

B. UX

C. UV

D. XY

 

Q. 9 A series is given, With one term missing. Choose the correct alternative from the given ones that will complete the series.

2,7, 14,2:,,34,?

A. 47

B. 39

C. 42

D. 46

 

Q. 10 A and B are standing at place “P”. They start moving in the opposite directions at the speed of 5 kmph and 4 kmph respectively. What will be the distance between them after 3 hours?

A. 3 km

B. 21 km

C. 18 km

D. 27 km

 

Q. 11 If Usha is taller than Nisha; Nisha is taller than Asha; Aika is taller then Usha; Harsha is shorter than ‘Asha; then who among them is the tallest?

A. Usha

B. Alka

C. Nisha

D. Asha

 

Q. 12 From the given alternative words, select the word which cannot be formed using the letters Of the given word :

DHARAMSALA

A. MASALA

B. ARAMANA

C. RAMA

D. SAHARA

 

Q. 13 If in a code GONE is written as ILPB then how may CRIB be written in that code?

A. EUKY

B. EKUY

C. EYUK

D. EOKY

 

Q. 14 In a certain code language,@ represents +, + represents -, α represents ‘+’ and – represents ‘x’. Find out the answer to the following question .

101 – 3 + 64 α 8 + 2 – 9 = ?

A. 295

B. 290

C. 209

D. 105

 

Q. 15 If 12 X 16 = 188 and 14 X 18 = 248, then find . the value of 16 x 20 =?

A. 320

B. 360

C. 316

D. 318

 

Q. 16 Find the missing number from the given alternatives

A. 60

B. 62

C. 64

D. 66

 

Q. 17 One morning, Raju walked towards the sun. After some time he turned left and again to his left. Which direction is he facing?

A. north

B. south

C. east

D. west

 

Q. 18 One or two statements are given followed by two conclusions/ assumptions, I and II.’You have to consider the statement to be true, even if it seems to be at variance from commonly known facts. You are to decide which of the given  conclusions/assumptions can definitely be drawn from the given statement. Indicate your answer.

Statement : If people are intelligent, they should be creative.

Conclusions/Assumptions:

I. Creativity and intelligence are related.

II. Creative people are intelligent

A. Only Conclusion I follows

B. Only Conclusion II follows

C. Both Conclusion I and Conclusion II follow

D. Neither Conclusion I nor Conclusion II follows

 

Q. 19 How many rectangles are there in the given figure?

A. 8

B. 15

C. 24

D. 30

 

Q. 20 In a village some of the gold-smiths are literates. Which diagram shows literate goldsmiths?

A. 1

B. 2

C. 3

D. 4

 

Q. 21  Which answer figure will complete the pattern in the question figure?

A. 1

B. 2

C. 3

D. 4

 

Q. 22 From the given answer figures, select the one in which the question figure is hidden/embedded.

A. 1

B. 2

C. 3

D. 4

 

Q. 23 A piece of paper is folded and cut as shown below in the question figures. From the given answer figures, indicate how it will appear when opened.

A. 1

B. 2

C. 3

D. 4

 

Q. 24 If a mirror is placed on the line MN, then which of the answer figures fs the right image of the given figure?

A. 1

B. 2

C. 3

D. 4

 

Q. 25 In this question, a word is represented by only one set of numbers as given in any one of the alternatives.The sets of numbers given in the alternatives are represented by two classes of alphabet as in two matrices given below. The columns and rows of Matrix I are numbered from O to 4 and that of Matrix II are numbered from 5 to 9. A letter from these matrices can be represented first by its row and next by its column e.g., O can be represented by 03, 11, etc., and ‘F’ can be represented by 55, 68, etc. Similarly you have to identify the set for the word ‘BEAD’

A. 97, 32, 14, 56

B. 88, 41, 20, 57

C. 57, 32, 41, 87

D. 75, 14, 20, 57

 

Q. 26 Who among the following Sikh Gurus had laid the foundation of Amritsar?

A. Guru Amar Das

B. Guru Ram Das

C. Guru Arjan Dev

D. Guru Har Govind

 

Q. 27 In the 42nd Constitutional Amendment 1976, which word was added to the Preamble?

A. Sovereign

B. Equality

C. Secular

D. Socialist

 

Q. 28 Which monument is known as “The National Monument of India”?

A. India Gate

B. Gateway of India

C. Raj Ghat

D. Red fort

 

Q. 29 Which of the following country has recently declared 3 month emergency following a failed military coup?

A. Sudan

B. Turkey

C. Maldives

D. Syria

 

Q. 30 The award which, the famous writer and social of activist Mahasweta Devi, who passed away recently, did not win

A. Sahity Akademi Award

B. Jnanpith Award

C. Saraswati Samman

D. Padam Vibhushan

 

Q. 31 Which place is said to be the Manchester of South india?

A. Coimbatore

B. Salem

C. Thanjavur

D. Madurai

 

Q. 32 A galvanometer can be converted into a voltmeter by connecting with it a

A. high resistance in parallel

B. low resistance on parallel

C. high resistance on Series

D. low resistance in series

 

Q. 33 In computer processing, __ selects processes from the pool and loads them into memory for execution.

A. Job Scheduler

B. Resource Scheduler

C. CPU Scheduler

D. Process Scheduler

 

Q. 34 Name the first cricketer to score- 1000 runs in an innings in any competitive match

A. Prithvi Shaw

B. Pranav Dhanawade

C. Virat Kohli

D. Shikhar Dhawan

 

Q. 35 The beach sands of Kerala are rich in

A. calcium

B. radium

C. thorium

D. manganese

 

Q. 36 During which of the following operating conditions of an automobile, carbon monoxide content in exhaust gas is maximum?

A. Acceleration

B. Cruising

C. Idle running

D. Deacceleration

 

Q. 37 India’s first Railway University will come up at

A. Vadodara, Gujarat

B. Bengaluru, Karnataka

C. Hyderabad,Andhra Pradesh

D. Lucknow, Uttar Pradesh

 

Q. 38 The Himalayas is the example of

A. Fold mountains

B. Block mountains

C. Ancient mountains

D. Residual mountains

 

Q. 39 The largest irrigation canal in India is____

A. Yamuna canal

B. Indira Gandhi canal

C. Sirhand canal

D. Upper Bari Doab canal

 

Q. 40 Even after sunset, the air near the Earth’s surface continue to receive heat due to

A. insolation

B. Terrestrial Radiation

C. Conduction

D. Convection

 

Q. 41 At the Rio Olympics, who was the flagbearer of the India contingent?

A. Narsingh Yadav

B. Abhinav Bindra

C. Dipa Karunakar

D. Sania Mirza

 

Q. 42 A landscape which is caused due to the fissure in the earth along which one side has moved down with reference to the other is known as.

A. Rift Valley

B. U Shaped Valley

C. V Shaped Valley

D. Hanging Valley

 

Q. 43 The largest artery in human body is

A. Aorta

B. Capillary

C. Vena cava

D. Pulmonary vein

 

Q. 44 An eudiometer measures

A. Atmospheric pressure

B. Time

C. Volume of gases

D. Vapour pressure

 

Q. 45 Trinitrotoluene is

A. used to melt metals

B. used to fuse two metals

C. used as an abrasive

D. used as an explosive

 

Q. 46 In a cut motion, when the amount of demand is reduced by Rs. 100 it is known as

A. Disapproval of policy cut

B. Economy cut

C. Vote on Account

D. Token cut

 

Q. 47 One of the leading producers of asbestos in the world is

A. Australia

B. Russia

C. Canada

D. Armenia

 

Q. 48 Beighton Cup t.s associated With which of the following

A. Cricket

B. Hockey

C. Football

D. Volleyball

 

Q. 49 If Reserve Bank of India reduces the cash reserve ratio, it will:

A. increase credit creation

B. decrease credit creation

C. have no impact on credit creation

D. have no definite impact on credit creation

 

Q. 50 Nitrification is the biological process of converting

A. N2 into nitrate

B. N2 into nitrite

C. Ammonia into nitrite

D. Ammonia into N2

 

Q. 51 20 men working 8 hours per day can complete a piece of work in 21 days. How many hours per day must 48 men work to complete the same job in 7 days?

A. 12

B. 20

C. 10

D. 15

 

Q. 52 ABCD is a cyclic quadrilateral ∠DBA = 50° and ∠ADB = 33°. Then the measure of ∠BCD is 

A. 83°

B. 80°

C. 75°

D. 60°

 

Q. 53 A shopkeeper earns a profit of 12% on selling a book at 10% discount on the printed price. The ratio of the cost price to the printed price of the book is

A. 45: 56

B. 50: 61

C. 99: 125

D. none of these

 

Q. 54 The number of pupils of a class is 55. The ratio of the number of male pupils to: the number of female pupils is 5: 6. The 11 number of female pupils is

A. 11

B. 25

C. 30

D. 35

 

Q. 55 5% more is gained by selling a watch for Rs. 350 than by selling it for Rs. 340. The cost price of the watch is

A. Rs. 110

B. Rs. 140

C. Rs. 200

D. Rs. 250

 

Q. 56 If 60% of the students in a school are boys and the number of girls is 812, how many boys are there in the school?

A. 1128

B. 1218

C. 1821

D. 1281

 

Q. 57 It takes eight hours for a 600 km journey, if 120 km is done by train and ,the rest by car. It takes 20 minutes more, if 200 km is done; by train and the rest by car. The ratio of the speed of the train to that of the car is:

A. 3:5

B. 3:4

C. 4:3

D. 4:5

 

Q. 58 If α+1/α =1. then value of (α²-α+1)/(α²+α+1) is (α≠0)

A. 1

B. -1

C. 0

D. 2

 

Q. 59 If m + n = 1, then the value of m³ + n³ + 3mn is: equal to

A. 0

B. 1

C. 2

D. 3

 

Q. 60 The maximum number of common tangents that can be drawn to two disjoint circles is

A. 1

B. 2

C. 4

D. Infinitely many

 

Q. 61 In figure, DE || BC. If DE = 3 cm, BC = 6 cm and area of ΔADE = 15 sq. cm, then the area of MBC is

A. 75 sq cm

B. 45 sq cm

C. 30 sq cm

D. 60 sq cm

 

Q. 62 If cos^4θ-sin^4θ=1/3, then the value of tan²θ is

A. 1/2

B. 1/3

C. 1/4

D. 1/5

 

Q. 63 If a perfect square, nQt divisible by 6, be divided by 6, the remainder will be

A. 1,3 or 5

B. 1, 2 or 5

C. 1, 3 or 4

D. 1, 2 or 4

 

Q. 64 A batsman in his 12th innings makes a score of 120, and thereby increases his average by 5. The average score after 12th innings is

A. 60

B. 55

C. 65

D. 70

 

Q. 65 The value of the root from figure is

A. 2

B. 4

C. ±2

D. -2

Q. 66 If x^4+1/x^4=119, then the value of (x-1/x) is

A. 6

B. 12

C. 11

D. 3

 

Q. 67 The side BC of MBC is extended to the point D. If ∠ACD =112° and ∠B = 3/4 ∠A.then the value of ∠B is

A. 64°

B. 48°

C. 46°

D. 50°

 

Q. 68 MBC is a right angled triangle, the radius of its circumcircle is 3 cm and the length of .Us altitude drawn from the opposite vertex to the hypotenuse is 2 cm. Then the area of the triangle is

A. 12 sq.cm

B. 3 sq.cm

C. 6 sq.cm

D. 5 sq.cm

 

Q. 69 The height of a tower is 50√3 metre. The angle of elevation of a tower from a distance 50 metre from its foot is

A. 30°

B. 45°

C. 60°

D. 90°

 

Q. 70 The amount of Rs. 10,000 after 2 years, compounded annually with the rate of interest being 10% per annum during the first year and 12% per annum during the second year, would be (in rupees)

A. 11,320

B. 12,000

C. 12,320

D. 12,500

 

Q. 71 The value of tan80° tan 10° +sin² 70° + sin² 20° is

A. 0

B. 1

C. 2

D. √3/2

 

Question:- 72-77

The bar graph given below shows the per acre yield (in kg) of different countries.

Study the graph carefully and answer the questions.

 

Q. 72 The average yield of the given countries is 

A. 132 1/3 kg

B. 133 1/3 kg

C. 134 1/3 kg

D. 135 1/3 kg

 

Q. 73 By how much percent is India’s per acre yield more than that of Pa!Q.stan’s?

A. 20%

B. 25%

C. 33 1/3 %

D. 35%

 

Q. 74 Sri Lanka’s yield {approximately) is what percent of total yield of all the countries?

A. 17.8%

B. 16.2%

C. 18.2%

D. 15.4%

 

Q. 75 Writing the yields of all countries in ascending order, the difference between the sum of yields of first three countries to that of last three countries is

A. 200 kg

B. 212 kg

C. 172 kg

D. 162 kg

 

Q. 76 In the following question, out of the four alternatives, choose the word which best expresses the meaning of the given word.CUPIDITY

A. fear

B. friendship

C. greed

D. love

 

Q. 77 In the following question, out of the four alternatives, choose the word which is opposite in meaning to the given word.

CAPTIVATE

A. Distract

B. Obscure

C. imprison

D. Release

 

Q. 78 Four words are given, out of which only one word is spelt correctly. Choose the correctly spelt word

A. Clandistine

B. Clandestine

C. Clandistene

D. Clandestene

 

Question:- 79-81

In the following questions, one part of the sentence may have- an error. Find out which part of the sentence has an error and choose the option corresponding to it. If the sentence is free from error, choose the “No Error” option.

Q. 79 Can I have (1)/ a loaf of bread (2)/ and a jam jar? (3)/ No error (4)

A. 1

B. 2

C. 3

D. 4

 

Q. 80 Now that I am back at work,(1)/ I have beginning (2)/ to feel much better. (3)/ No error (4)

A. 1

B. 2

C. 3

D. 4

 

Q. 81 The artist, plainly a better critic (1)/ than painter, destroyed what (2)/ he made over for ten years. (3)/ No error (4)

A. 1

B. 2

C. 3

D. 4

 

Question :- 82-84

In the following questions, the sentences given with blanks are to be filled With an appropriate word(s). Four alternatives are suggested for each question. For each question, choose the correct alternative

 

Q. 82 Never give your friends _

A. the cold arm

B. the cold elbow

C. the cold shoulder

D. the cold hand

 

Q. 83 My mother upset the kettle of boiling water. and __ her hand.

A. scalded

B. scolded

C. scorched

D. wounded

 

Q. 84 His Writings are._ mistakes

A. brooded with

B. burst into

C. replete with

D. boasted of

 

Question :- 85-87

In each of the questions, four alternatives are given for the Idiom/Phrase. Choose the alternative which best expresses the meaning of the Idiom/Phrase

 

Q. 85 cat a sorry figure

A. Did not stand straight

B. Apologised for his remarks

C. Created a wrong impression

D. Made a poor impression 

 

Q. 86 To take to task

A. Forgave him

B. Slapped him

C. Gave him extra work

D. Reprimanded him

 

Q. 87 Bring to light

A. Introduced

B. Revealed

C. Seen

D. Brought to life

 

Question:- 88-90

Ouf of the four alternatives choose the one which can be substituted for the given words/sentences.

 

Q. 88 A job carrying no salary

A. Honorory

B. Memento

C. Honorarium

D. Memorandium

 

Q. 89 Act of stealing something in small quantities

A. Pillage

B. Plagiarise

C. Proliferate

D. Pilferage

 

Q. 90 Pertaining to the west

A. Celestial

B. Occidental

C. Oriental

D. Terrestrial

 

Question:-91-95

In each of the following questions, a sentence/a part of the sentence is printed in bold. Four alternatives are’ given to the bold part which will improve the sentence. Choose the correct alternative corresponding to it. In case no improvement is needed; choose the alternative corresponding to “No improvement”.

 

Q. 91 As soon as I arrived in home, l knew that something was wrong

A. arrived by home

B. arrived home

C. arrived my home

D. no improvement

 

Q. 92 Unfortunately I did not pass in the examination

A. I did not pass over

B. I did not overcome

C. I did not pass

D. No improvement

 

Q. 93 His powerful desire brought about his downfall

A. His wishful desire

B. His desire for power

C. His seager desire

D. no improvement

 

Q. 94 Ramesh laid in the shade of a tree before he could walk further

A. lied

B. lay

C. lain

D. no improvement

 

Q. 95 An education in handling money would Imply the ability to oversee, the consequences of overspending or over-borrowing.

A. foresee

B. overlook

C. overvalue

D. no improvement

 

Question :- 96-100

A passage is given with 5 questions following it. Read the passage carefully and choose the best answer to each question out of the four alternatives.

A knowledge of grammar is essential for good speaking and writing, by which one’s mind is judged ·Studying grammar means hard work : it must be learned as a whole with no part omitted. and it demands much thought and patience. But, once acquired, it can give a lifetime’s pleasure and profit. Its study requires no physical hardship, no special room or expenses. If people spent only their leisure time studying grammar they could master it in one year. The author learned it in less than a year. As a private soldier earning sixpence a day, he sat on his bed and studied. Unable to afford candle or oil; he read in winter by firelight (when it was his turn). If he could manage it thus, and with no outside encouragement, then any youth, however poor or busy, could do the same. 

 

Q. 96 Why should we learn grammar?

A. To develop speaking skills

B. To develop writing skills

C. To have a mastery over language

D. To acquire good speaking and writing skills

 

Q. 97 How does the world judge a man’s mind?

A. By his dress

B. By his manners

C. By his appearance

D. By his speech and writing

 

Q. 98 How long would it take to gain mastery over grammar

A. 1year

B. 6 months

C. 2 years

D. 10 months

 

Q. 99 What is the occupation of the writer?

A. teacher

B. soldier

C. artist

D. clerk

 

Q. 100 The learning of grammar should be

A. patient, thoughtful and holistic

B. thoughtful, patient and piecemeal

C. holistic, thoughtful and rapid

D. thoughtful, rapid and piecemeal

 

 

Answer Sheet 
Question 1 2 3 4 5 6 7 8 9 10
Answer B D B A D A D B A D
Question 11 12 13 14 15 16 17 18 19 20
Answer B B D A C C D C D B
Question 21 22 23 24 25 26 27 28 29 30
Answer A D D D C B A A B C
Question 31 32 33 34 35 36 37 38 39 40
Answer A C A B C C A A B B
Question 41 42 43 44 45 46 47 48 49 50
Answer B A A C D D B B A C
Question 51 52 53 54 55 56 57 58 59 60
Answer C A A C C B B C B C
Question 61 62 63 64 65 66 67 68 69 70
Answer D A C C A D B C C C
Question 71 72 73 74 75 76 77 78 79 80
Answer C D B B C C A B C B
Question 81 82 83 84 85 86 87 88 89 90
Answer C C A C C D B A D B
Question 91 92 93 94 95 96 97 98 99 100
Answer B C B B A D D A B A

 

SSC CGL Tier-I 02 September 2016 Shift-II Previous Year Paper

SSC CGL Tier 1 2016 2 Sep Shift 2

Q. 1 Select the related word from the given alternatives.

Cell : Cytology :: Birds : ?

A. Odontology

B. Mycology

C. Ornithology

D. Etymology

 

Q. 2 Select the related letters from the given alternatives.

DHLP : WSOK :: FJNR : ?

A. UQMI

B. TPLH

C. SOKG

D. VRNJ

 

Q. 3 Select the related number from the given alternatives.’

2 : 10 :: 26 : ?

A. 50

B. 36

C. 42

D. 20

 

Q. 4 Find the odd letters from the given alternatives.

A. EHJ

B. JML

C. PSR

D. VYX

 

Q. 5 Find the odd word from the given alternatives.

A. Tomcat

B. Baby

C. Fawn

D. Cub

 

Q. 6 Find the odd number from the given alternatives.

A. 11

B. 17

C. 19

D. 21

 

Q. 7 Arrange the following words as per order in the dictionary.

1. Patriot

2. Pastor

3. Patron

4. Pattern

5. Pastern

A. 5, 2, 3, 1, 4

B. 5, 2, 1, 3, 4

C. 5, 2, 4, 1 ,3

D. 5, 2, 4, 3, 1

 

Q. 8 A series is given below with one term missing. Choose the correct alternative from the given ones that will complete the series.

SCD, TEF, UGH, ? , WKL

A. CMN

B. VJI

C. VIJ

D. IJT

 

Q. 9 A series is given below with one term missing. Choose the correct alternative from the given ones that will complete the series.

124, 235, 346, 457, ?

A. 455

B. 465

C. 565

D. 568

 

Q. 10 F is the daughter of A. C is the daughter of A. K is the sister of F, G is the brother of C. Who is the uncle of G?

A. A

B. C

C. K

D. F

 

Q. 11 If Jhansi is 12 ahead in rank of Prabha, who ranks 15th from last, then how many students are there in the class if Jhansi ranks 4th in order of merit?

A. 23

B. 27

C. 30

D. 31

 

Q. 12 From the given words, select the word which cannot be formed using the letters of the given word.

ENVIRONMENT

A. ENTER

B. METRE

C. IRON

D. MOMENTUM

 

Q. 13 If ‘BASKET’ is written as ‘TEKSAB’ how can ‘PILLOW’ be written in that code?

A. LOWPIL

B. WOLLIP

C. LOWLIP

D. WOLPIL

 

Q. 14 If ‘+’ means ‘-‘ and ‘-‘ means ‘x’, ‘x’ means ‘÷’ and ‘÷’ means ‘+’ then 2 ÷ 6 x 6 ÷ 2 = ?

A. 1

B. 0

C. 10

D. 5

 

Q. 15 If 6 x 9 x 3 = 963 and 4 x 8 x 5 = 845, then 9 x 4 x 7 = ?

A. 974

B. 479

C. 497

D. 749

 

Q. 16 Select the missing number in figure (1) from the given responses:

A. 4

B. 3

C. 2

D. 1

 

Q. 17 Ashok went 8 kms south and turned west and walked 3 kms. Again he turned north and walked 5 kms. He took a final turn to east and walked 3 kms. In which direction was Ashok from the starting point?

A. East

B. North

C. West

D. South

 

Q. 18 Consider the given statement/s to be true and decide which of the given conclusions/assumptions can definitely be drawn from the given statement. Statements: Ahimsa should be basic principle of human beings. All of us should practise Ahimsa.

Conclusions:

I. We all know the real meaning of Ahimsa.

II. Ahimsa promotes global harmony.

A. Only Conclusion I follows

B. Only Conclusion II follows

C. Neither I Conclusion nor II follows

D. Both conclusion I and II follow

 

Q. 19 Of a cubical container shown in figure (1), all the six faces have different markings indicating the position of the equipment packed inside. Which symbol is opposite the face having C?

A. Z

B. B

C. X

D. Y

 

Q. 20 Identify the diagram among (1), (2), (3), (4), that best represents the relationship among classes given below:

Minutes, Days, Months

 

A. (1)

B. (2)

C. (3)

D. (4)

 

Q. 21 Which answer figure among (1), (2), (3), (4) will complete the pattern in the question figure (A)?

 

A. (1)

B. (2)

C. (3)

D. (4)

 

Q. 22 From the given answer figures among (1), (2), (3), (4), select the one in which the question figure (A) is hidden/embedded.

A. (1)

B. (2)

C. (3)

D. (4)

 

Q. 23 A piece of paper is folded and cut as shown in Figure (A). From the given answer figures (1), (2), (3), (4), indicate how it will appear when opened.

 

A. (1)

B. (2)

C. (3)

D. (4)

 

Q. 24 If a mirror is placed on the line MN, then which of the answer figures (1), (2), (3), (4) is the right image of the given figure (A)?

A. (1)

B. (2)

C. (3)

D. (4)

 

Q. 25 In the question, a word is represented by only one set of numbers as given in any one of the alternatives. The sets of numbers given in the alternatives are represented by two classes of alphabets as in two matrices given in figure (1). The columns and rows of Matrix I are numbered from 0 to 4 and that of Matrix II are numbered from 5 to 9. A letter from these matrices can be represented first by its row and next by its column, e.g., G can be represented by 03, 12 etc, and ‘L’ can be represented by 57, 65, etc. Similarly you have to identify the set for the word ‘DATE’:

A. 23, 68, 24, 97

B. 14, 96, 11, 85

C. 23, 96, 40, 85

D. 32, 89, 10, 68

 

Q. 26 MUDRA Bank has been launched to help

A. Small business

B. Marginal farmers

C. Poor women

D. Rural sector

 

Q. 27 Which of the following controls the insurance business of India?

A. RBI

B. IDBI

C. SEBI

D. IRDA

 

Q. 28 The Speaker of Lok Sabha addresses his letter of resignation to the

A. President of India

B. Prime Minister

C. Deputy Speaker of Lok Sabha

D. The Chief Justice of India

 

Q. 29 Who presided over the first session of the Indian National Congress?

A. A. O. Hume

B. Surendranath Banerjee

C. W. C. Banerjee

D. Badruddin Tayyabji

 

Q. 30 Which of the following writers has called Akbar’s Din-i-Ilahi as a monument of his folly, not of wisdom?

A. Badayuni

B. Vincent Smith

C. Baroi

D. W. Haig

 

Q. 31 Which of the following region in India is now regarded as an ‘Ecological Hot Spot’?

A. Western Himalayas

B. Eastern Himalayas

C. Western Ghats

D. Eastern Ghats

 

Q. 32 The Beaufort scale is used to measure:

A. Atmospheric pressure

B. Altitudes of mountains

C. Wind velocity

D. Intensity of earthquakes

 

Q. 33 The waste management technique that involves the use of micro-organisms to remove or neutralize pollutants from contaminated site is called

A. Bio sensor

B. Bio magnification

C. Bio remediation

D. Bio concentration

 

Q. 34 In IT networking, which of the following device is used in physical layer?

A. Repeater

B. Router

C. Transport Gateway

D. Bridge

 

Q. 35 The purest form of water in nature is:

A. Rain water

B. Lake water

C. River water

D. Sea water

 

Q. 36 The source of energy that causes the least global warming is

A. Coal

B. Geothermal energy

C. Natural Gas

D. Petroleum

 

Q. 37 The constellation ‘Sapta Rishi’ is known to Westerners as the

A. Seven Monk

B. Alpha Centauri

C. Big Dipper

D. Small Bear

 

Q. 38 Bridge is the technique used in

A. Athletics

B. Wrestling

C. Weight lifting

D. Karate

 

Q. 39 Which of the following books is called the ‘Bible of Socialism’?

A. Economics of Welfare

B. Das Capital

C. Value and Capital

D. Asian Drama

 

Q. 40 Which country recently detonated its first hydrogen bomb?

A. North Korea

B. South Korea

C. Iran

D. Libya

 

Q. 41 The Swarna Jayanti Shahari Rojgar Yojana (SJSRY) mainly aims at creating employment opportunities for

A. both self employment and wage employment in urban areas

B. self employment in urban areas only

C. wage employment in urban areas only

D. None of these

 

Q. 42 Who is known as the ‘Father of Green Revolution’ in India?

A. G. Paul

B. M. S. Swaminathan

C. Van Neil

D. Dr. Mithchell

 

Q. 43 The component used for tuning a radio is basically a variable

A. Resistor

B. Condenser

C. Inductor

D. Transformer

 

Q. 44 Which of the following is not a donor atom?

A. Phosphorous

B. Antimony

C. Arsenic

D. Aluminium

 

Q. 45 Which one of the following party was founded by Subhash Chandra Bose?

A. Abhinav Bharat

B. Azad Hind Sena

C. Revolutionary Army

D. Forward Block

 

Q. 46 0°K is equivalent to

A. 273°C

B. -273°C

C. 0°C

D. 100°C

 

Q. 47 The oxygen liberated during photosynthesis comes from

A. Water

B. Carbon dioxide

C. Glucose

D. Chlorophyll

 

Q. 48 First national park of India that was established in 1936 was named as

A. Kanha National Park

B. Bharatpur National Park

C. Hailey National Park

D. Rajaji National Park

 

Q. 49 Who is the 1st Indian female amputee to climb Mount Everest?

A. Arunima Sinha

B. Bachendri Pal

C. Santosh Yadav

D. Premlata Agarwal

 

Q. 50 The term “Higgs Boson” is associated with

A. Nano Technology

B. Oncology

C. God Particle

D. Stem Cell Research

 

Q. 51 If 100 cats kill 100 mice in 100 days, then 4 cats would kill 4 mice in how many days?

A. 4 days

B. 3 days

C. 40 days

D. 100 days

 

Q. 52 In Δ ABC, DE || BC, such that AD/BD = 3/5. If AC = 5.6 cm, then AE is equal to

A. 4.2 cm

B. 3.1 cm

C. 2.8 cm

D. 2.1 cm

 

Q. 53 Two circles touch each other internally. The greater circle has its radius is 6 cm and the distance between the centres of the circles is 2 cm. The radius of the other circle is

A. 3 cm

B. 4 cm

C. 2 cm

D. 5 cm

 

Q. 54 An office opens at 10 AM and closes at 5 PM. The lunch interval is for 30 minutes. The ratio of lunch interval to the total period of office hours is

A. 1 : 7

B. 1 : 14

C. 7 : 1

D. 14 : 1

 

Q. 55 A bookseller allowed 15% discount on the books sold. Sunil purchased books worth Rs. 1500. How much will he have to pay to the bookseller.

A. Rs. 1200

B. Rs. 1250

C. Rs. 1275

D. Rs. 1300

 

Q. 56 If the ratio between the profit and selling price of an article is 1 : 5, then the ratio between the selling price and the cost price of that article is:

A. 3 : 2

B. 4 : 3

C. 5 : 4

D. 6 : 5

 

Q. 57 What percent of 1 day is 36 minutes?

A. 25%

B. 2.5%

C. 3.6%

D. 0.25%

 

Q. 58 A car covers a certain distance in 25 hours. If it reduces the speed by 1/5th, the car covers 200 km less in that time. The speed of car is

A. 60 km/hr

B. 30 km/hr

C. 40 km/hr

D. 50 km/hr

 

Q. 59 At what percent of simple interest will a sum of money double itself in 15 years?

A. 6 1/3%

B. 6 2/3%

C. 6 1/2%

D. 6%

 

Q. 60 1/√a – 1/√b = 0, then the value of 1/a + 1/b is :

A. 1√ab

B. √ab

C. 2/√ab

D. 1/2√ab

 

Q. 61 If x^2 + y^2 + z^2 = 14 and xy + yz + zx = 11, then the value of (x + y + z)^2 is

A. 16

B. 25

C. 36

D. 49

 

Q. 62 If √2 tan 2θ = √6 and 0° < θ < 45°, then the value of sinθ + √3 cosθ – 2tan^2θ is

A. 2/3

B. 4/3

C. 2

D. 8/3

 

Q. 63 A positive number when decreased by 4, is equal to 21 times the reciprocal of this number. 

This number is:

A. 3

B. 7

C. 5

D. 9

 

Q. 64 If tan α = 2, then the value of (sin α) / (sin^3 α + cos^3 α) is

A. 2/9

B. √5/9

C. 10/9

D. 5√5/9

 

Q. 65 If x = ∛28, y = ∛27, then the value of x + y – 1/(x^2 + xy + y^2) is

A. 8

B. 7

C. 6

D. 5

 

Q. 66 If x = 12 and y = 4, then the value of (x + y)^x/y is

A. 48

B. 1792

C. 4096

D. 570

 

Q. 67 In a triangle PQR, PQ = PR and ∠Q is twice that of ∠P. Then ∠Q is equal to

A. 72°

B. 36°

C. 144°

D. 108°

 

Q. 68 If the length of a chord of a circle is 16 cm and is at a distance of 15 cm from the centre of the circle, then the radius of the circle (in cm) is:

A. 15

B. 16

C. 17

D. 34

 

Q. 69 On 24th May, 2008 the maximum temperature of Delhi, Kolkata and Mumbai were recorded as 35°C, 33°C and 34°c respectively. What was the maximum temperature of Chennai so that the average maximum temperature of those cities would be 35°C?

A. 34°C

B. 35°C

C. 36°C

D. 38°C

 

Q. 70 The diameter of a sphere is twice the diameter of another sphere. The curved surface area of the first and the volume of the second are numerically equal. The numerical value of the radius of the first sphere is

A. 3

B. 24

C. 8

D. 16

 

Q. 71 The thread of a kite makes angle 60° with the horizontal plane. If the length of the thread be 80 m, then the vertical height of the kite will be

A. 40/√3 metre

B. 80√3 metre

C. 80 metre

D. 40√3 metre

 

Question:- 72-78

Study the following pie chart given in figure (1) carefully and answer the questions. The pie chart represents the percentage of people involved in various occupations.

 

Q. 72 How many more people are involved in service than in trade?

A. 3660

B. 2660

C. 1660

D. 660

 

Q. 73 The ratio of the people involved in service to that in industry is

A. 1 : 2

B. 2 : 3

C. 3 : 4

D. 3 : 2

 

Q. 74 The sectoral angle made by the people involved in service in the given pie-chart is 

A. 36°

B. 90°

C. 72°

D. 108°

 

Q. 75 The difference between the maximum number of people involved and minimum number of people involved in various professions is

A. 2640

B. 3640

C. 6320

D. 5320

 

Q. 76 In the following question, out of the four alternatives, choose the word which best expresses the meaning of the given word.

LABYRINTH

A. Maze

B. Path

C. Skyscraper

D. Impasse

 

Q. 77 In the following question, out of the four alternatives, choose the word which is opposite in meaning to the given word.

BARREN

A. Fertile

B. Abundant

C. Harsh

D. Fallow

 

Q. 78 Four words are given, out of which only one word is spelt correctly. Choose the correctly spelt word.

A. Mysogynous

B. Misogynous

C. Mysoginous

D. Misoginous

 

Question:- 79-81

In the following questions, one part of the sentence may have an error. Find out which part of the sentence has an error. If the sentence is free from error, choose “No error”:

 

Q. 79 According to scientists (1)/ there are a lot of (2)/ answers about it. (3)/ No error (4).

A. (1)

B. (2)

C. (3)

D. (4)

 

Q. 80 They are coming (1)/ straight to (2)/ our direction. (3)/ No error.(4)

A. (1)

B. (2)

C. (3)

D. (4)

 

Q. 81 The building collapsed (1)/ at the afternoon (2)/ at about 4 o’ clock. (3)/ No error (4).

A. (1)

B. (2)

C. (3)

D. (4)

 

Question:- 82-84

In the following questions, the sentences given with blanks are to be filled with an appropriate word(s). Four alternatives are suggested for each question. 

 

Q. 82 The police is ________________________ on his activities.

A. keeping an eye

B. keeping watchful eyes

C. keeping both eyes

D. keeping on eye

 

Q. 83 Are you ______________________________ stamp collecting?

A. interested by

B. interested in

C. interested at

D. interested about

 

Q. 84 Your report _____________________ my statement.

A. conforms

B. conforming

C. confirms

D. comforting

 

Question:- 85-87

In each of the questions, four alternatives are given for the Idiom/Phrase.Choose the alternative which best expresses the meaning of the Idiom/Phrase. 

 

Q. 85 To get cold feet

A. Drenched

B. Fear

C. Felicitate

D. Fever

 

Q. 86 Beside the mark

A. Perfect

B. Not to the point

C. Relevant

D. Charming

 

Q. 87 On tenterhooks

A. Happy and excited

B. In suspense and anxiety

C. Angry and irritated

D. Surprised and shocked

 

Question:-88-90

In the following questions, out of the four alternatives, chose the one which can be substituted for the given words/sentences. 

 

Q. 88 Property inherited from one’s father or ancestors

A. Patrimony

B. Mercenary

C. Hereditary

D. Aristocracy

 

Q. 89 A person who is womanish in his habits

A. Feminist

B. Philogynist

C. Effeminate

D. Feminine

 

Q. 90 One who is converted from one religion to another

A. Pilgrim

B. Polytheist

C. Proselyte

D. Presbyte

 

Question :- 91 – 95

In each of the following questions, a sentence/a part of the sentence is quoted. Four alternatives are given to the quoted part which will improve the sentence. Choose the correct alternative. In case no improvement is needed, choose “no improvement”.

 

Q. 91 Do you have “some sugar”?

A. any sugar

B. little sugar

C. small sugar

D. no improvement

 

Q. 92 Iago “lied” to Othello about his wife.

A. Lay

B. laid

C. lain

D. No improvement

 

Q. 93 It is not possible to tell the

entire story “in nutshell”.

A. in a nutshell

B. in the nut

C. in a shell

D. No improvement

 

Q. 94 It is “all but same” to me whether I am transferred to Mumbai or Kolkata.

A. all or same

B. all the same

C. all one

D. no improvement

 

Q. 95 Despite his father’s financial assistance he was always “hard on”

A. hard in

B. hard up

C. hard out

D. no improvement

 

Questions: 96 – 100

Even the majority of elders turn their homes into hives of worry as they have too little to do in too much time. Those who have retired this find retirement tiresome when hobbies, instead, could have turned it into a period of creativity and contentment. This common problem of inability to utilise leisure pleasurably and profitably is not restricted to Indians. In fact, Japanese are the worst sufferers. Their weekends, rather than increase their enjoyments of life, have wreaked havoc on their health and happiness. Unable to while away the long, unstructured hours, many of them have become addicts to coffee or hard liquor, and have even taken to gambling. How has this social malady come about? Ironically, the syllabus-loaded education system is the main culprit. It places a heavy word-load on children and youth, laying emphasis as it does on memory rather than intelligence. 

 

Q. 96 Why do a majority of retired elders find retirement tiresome?

A. Because their homes have been turned into hives of worry.

B. Because they do not have hobbies to utilise their free time

C. Because they had nothing to do.

D. Because they had plenty of free time

 

Q. 97 How have the Japanese benefited from their weekends?

A. They have increased enjoyment in life

B. They enjoy health and happiness

C. They use their free time to increase their enjoyment

D. They become addicts to coffee or hard liquor and gambling.

 

Q. 98 The syllabus-loaded education system

A. Places a heavy burden on the youth

B. Ensures that parents pay attention to the development of children  

C. Lays emphasis on intelligence

D. Gives students a lot of free time

 

Q. 99 The author thinks that

A. Authorities are more appreciative of the syllabus-loaded education system.

B. Hobbies play an important role in changing the unfortunate situation.

C. Co Curricular activities are discouraged

D. Only Indians suffer from inability to utilise leisure

 

Q. 100 The passage tells us that

A. Hobbies are a waste of time

B. Hobbies play a crucial role in physical and mental development

C. Hobbies wreck havoc on man’s health and happiness

D. Hobbies can turn us into addicts of coffee, liquor or gambling

 

 

Answer Sheet 
Question 1 2 3 4 5 6 7 8 9 10
Answer C A A A B D B C D D
Question 11 12 13 14 15 16 17 18 19 20
Answer C D B D C C D C A A
Question 21 22 23 24 25 26 27 28 29 30
Answer B A B C A A D C C B
Question 31 32 33 34 35 36 37 38 39 40
Answer C C C A A B C B B A
Question 41 42 43 44 45 46 47 48 49 50
Answer A B B D D B A C A C
Question 51 52 53 54 55 56 57 58 59 60
Answer D D B B C C B C B C
Question 61 62 63 64 65 66 67 68 69 70
Answer C B B C C C A C D B
Question 71 72 73 74 75 76 77 78 79 80
Answer D B B C D A A B C B
Question 81 82 83 84 85 86 87 88 89 90
Answer B A B C B B B A C C
Question 91 92 93 94 95 96 97 98 99 100
Answer A D A B B B D A B B

SSC CGL Tier-I 01 September 2016 Shift-II Previous Year Paper

SSC CGL Tier 1 2016 1 Sep Shift 2

Q. 1 Find the odd letters from the given alternatives : 

A. AEFJ

B. KOPT

C. UYZD

D. EHIL

 

Q. 2 Find the odd number-pair from the given alternatives :

A. 81 : 243

B. 16 : 64

C. 64 : 192

D. 25 : 75

 

Q. 3 Find the odd word from the given alternatives

A. Distinguish

B. Scatter

C. Differentiate

D. Classification

 

Q. 4 Arrange the following words as per the English dictionary and find the last word.

Leaf Less Lean Leave

A. Lean

B. Leave

C. Less

D. Leaf

 

Q. 5 How many triangles are there in the given figure?

A. 5

B. 7

C. 8

D. 9

 

Q. 6 In a certain code language, APPROACH is coded as CHOAPRAP. How will RESTRICT be coded?

A. CTRISTER

B. ERTSIRTC

C. CTRISTRE

D. TCIRSTRE

 

Q. 7 Select the related word from the given alternatives.

Apes : Gibber : : Camels : ?

A. grunt

B. cheep

C. bleat

D. whine

 

Q. 8 Select the related letters from the given alternatives

TSR : FED : : WVU : ?

A. CAB

B. MLK

C. PQS

D. GFH

 

Q. 9 Select the related number from the given alternatives.

7: 32:: 28:?

A. 126

B. 136

C. 116

D. 128

 

Q. 10 John, in the morning, started walking towards North and then turn towards opposite side of the sun. He then turns left again and stops. Which direction is he facing now?

A. north

B. west

C. south

D. east

 

Q. 11 If 64 + 7 = 460

43+8=?

25 + 8 = 212

A. 360

B. 376

C. 332

D. 356

 

Q. 12 In a village there are landlords of which some are literates. Which of the following best expresses the relationship between them?

A. 1

B. 2

C. 3

D. 4

 

Q. 13 Which one of the Answer figures completes the Question figure?

A. 1

B. 2

C. 3

D. 4

 

Q. 14 What is the mirror image of the following figure? Line AB represents the mirror.

A. 1

B. 2

C. 3

D. 4

 

Q. 15 If the given words are arranged in descending order, then which of the following be last?

Sapling Tree Plant Seed

A. sapling

B. plant

C. seed

D. tree

 

Q. 16 Which one is wrong number in the given series?

7 56 44 7 3584 28672 

A. 3584

B. 56

C. 7

D. 447

 

Q. 17 In this question, a statement is followed by two assumptions I and II. You have to consider the statements to be true even if they seem to be at variance from the commonly known facts. You have to decide which of the following assumptions logically follows from the given statement.

Statements : Only good singers are invited in the conference. No one without sweet voice is a good singer.

Assumption I : All invited singers in the conference have sweet voice.

Assumption II : Those singers who do not have sweet voice are not invited in the

conference

A. Only I follows

B. Neither I nor II follows

C. Both i and II follow

D. Only II follows

 

Q. 18 If’+’ means ‘x’, ‘-‘ means ‘+’, ‘x’ means’-‘ and’+’ means’+’, then find the value of the following equation.

6 + 64 – 8 + 45 x 8

A. 85

B. 76

C. 87

D. 75

 

Q. 19 Which of the following has the given figμre embedded in it?

A. 1

B. 2

C. 3

D. 4

 

Q. 20 Find the missing number from the given alternatives

1 4 2 3 2 ?

A. 2

B. 5

C. 3

D. 4

 

Q. 21 Find the missing number from the given alternatives

A. 13

B. 15

C. 17

D. 19

 

Q. 22 Find the missing letters from the given’ alternatives :

AEN MQZ CGP ?

A. OSB

B. PUE

C. MPX

D. OTC

 

Q. 23 Find the missing number from the given alternatives :

0 4 18 48 ? 180

A. 58

B. 68

C. 84

D. 100

 

Q. 24 Select the word which cannot be fortune4 using the letters of the given· word?

SEGREGATION

A. EAGER

B. SEA

C. GATE

D. NATION

 

Q. 25 A word is represented by only one set of rt numbers as given in any one of the alternatives. The sets of numbers given in the alternatives are represented by two classes of alphabet by two matrices given below. The columns and row of Matrix l are numbered from O to 4 and that of Matrix II are numbered from 5 to 9. A letter from these matrices can be represented first by its row and next by its column. For example, ‘U’ can be represented by 03, 14, 32 etc. and ‘O’ can be represented by 56, 67, 75 etc. Similarly you have to identify the set for the word given in the question. PURE 

A. 69, 14, 04, 98

B. 34, 76, 31, 79

C. 04, 32, 87, 59

D. 69, 99, 31, 01

 

Q. 26 A number when divided by 6 leaves remainder 3. When the square of the same number is divided by 6, the remainder is:

A. 0

B. 2

C. 1

D. 3

 

Q. 27 Find the wrong number in the following number series.

3 7 16 35 70 153

A. 70

B. 16

C. 153

D. 35

 

Q. 28 The averages of runs scored by a cricket player in 11 innings is 63 and the average of his first six innings is 60 and the av_erage of last six innings is 65. Find the runs scored by him in the sixth innings.

A. 60

B. 54

C. 67

D. 57

 

Q. 29 In a parade of school students, the number of boys and girls are in the ratio of 9 : 7 respectively and the number of students is 256. Find the number of girls.

A. 102

B. 112

C. 118

D. 128

 

Q. 30 Two persons ride towards each other from two places 55 km apart, one riding at 12 km hr and the other at 10 km/hr. In what time will they be 11 km apart?

A. 2 hours and 30 minutes

B. 1 hours and 30 minutes

C. 2 hours

D. 2 hours and 45 minutes

 

Q. 31 A train 150 m long passes a telegraphic post in 12 seconds. Find the speed of the train.(in km/hr)

A. 50

B. 12.5

C. 25

D. 45

 

Q. 32 In an election, a candidate secures 40% of the votes but is defeated by the only other candidate by a majority of 298 votes, Find the total number of votes recorded

A. 1580

B. 1490

C. 1470

D. 1530

 

Q. 33 If x-1/x=2, then what is the value of x²+1/x² ?

A. 4

B. 5

C. 3

D. 6

 

Q. 34 The respective ratio between the height of tower and the point at some distance from its foot is 5√3 :5. What will be the angle of elevation of the top of the tower?

A. 30°

B. 60°

C. 90°

D. 45°

 

Q. 35 Successive discounts of 20% and 10% are given on an item marked at Rs.700 Find· the selling price

A. Rs. 504

B. Rs. 196

C. Rs. 582

D. Rs. 601

 

Q. 36 The population of a city is 20000. It increases by 20% during the first year and 30% during the second year. The population after two years will be:

A. 32000

B. 40000

C. 31200

D. 30000

 

Q. 37 A reduction of 20% in the price of rice enables a buyer to buy 5 kg more for rupees 1200. The reduced price per kg· of rice will be:

A. 36

B. 45

C. 48

D. 60

Q. 38 A man spends 60% of his income on different items. His income is increased by 20% and his expenditure is also increased by 10%. Find the percentage decrease in his savings?

A. 10%

B. 15%

C. 20%

D. 25%

 

Q. 39 The lengths of side AB and side BC of a scalene triangle ABC are 12 cm and 8cm respectively. The size of angle C is 90° Find the approximate length of side AC.

A. 12

B. 9

C. 14

D. 16

 

Q. 40 Find the value of (sin27°/cos63°)²+(cos63°/sin27°)²

A. 0

B. 2

C. 3

D. 1

 

Q. 41 The sum of the ages of mother and her daughter is 60 years. 12 years ago the mother was eight times as old as her daughter. How old is the daughter at present?

A. 20 years

B. 28 years

C. 16 years

D. 12 years

 

Q. 42 A boat goes at 14 kmph along the stream and 8 kmph against the stream. The speed of the boat (in kmph) in still water is:

A. 12 kmph

B. 11 kmph

C. 10 kmph

D. 8 kmph

 

Q. 43 A shopkeeper buys 80 articles for Rs. 2400 and sells them for a profit of 16%. Find the selling price of one article

A. Rs. 36.40

B. Rs. 34.80

C. Rs. 35.60

D. Rs. 33.80

 

Q. 44 Find the amount which Shyam will get on Rs. 4096, if he gives it for 18 months at 12 ½ % per annum, interest being compounded half yearly

A. Rs. 5,813

B. Rs. 4,515

C. Rs. 4,913

D. Rs. 5,713

 

Q. 45 A works twice as fast as B. If B can complete a piece of work independently in 12 days, then what will be the number of days taken by A and _B together to finish the work?

A. 4

B. 6

C. 8

D. 18

 

 

Q. 46 The foreign exchange reserve in 2012 was how many times that in 2009?

A. 0.7

B. 1.2

C. 1.4

D. 1.5

 

Q. 47 What was the percentage increase in the foreign reserves in 2012 over 2008?

A. 100

B. 150

C. 200

D. 620

 

Q. 48 The ratio of the number of years, in which the foreign exchange reserves are above the average reserves, to those in which reserves are below the average reserves, is

A. 2:6

B. 3:4

C. 3:5

D. 1:1

 

Q. 49 A well of diameter 3m is dug 14m deep. The earth taken out of it has beens read evenly all around it in the shape of a circular ring of width 4m to form an embankment. Find the height of the embankment

A. 4.25m

B. 2.25m

C. 1.125m

D. 1. 75m

 

Q. 50 If x³+1/x³ =110, then find the value of x+1/x

A. 2

B. 3

C. 4

D. 5

 

Question :- 51-54

In each of the following questions, choose the word which is most similar in meaning to the given word.

 

Q. 51 DISHEVELLED

A. tidy

B. clean

C. neat

D. untidy

 

Q. 52 choose the correct option

VENERATE

A. despise

B. disobey

C. disregard

D. reverse

 

Question:- 53-54

In each of the following questions, choose the word which is most opposite in meaning to the given word.

 

Q. 53 choose the correct option

CONGENIAL

A. accord

B. snug

C. engaging

D. unpleasant

 

Q. 54 choose the correct option

ABJURE

A. renounce

B. relinquish

C. abnegate

D. acquire

 

Question :- 55-57

In the following questions, out of the four alternatives choose the one which can be substituted for the given words/sentence.

 

Q. 55 highly skilled

A. consummate

B. inveterate

C. notorious

D. maladroit

 

Q. 56 identification with the feelings of another

A. sympathy

B. empathy

C. apathy

D. compassion

 

Q. 57 insatiable desire for wealth

A. selfish

B. avarice

C. egoist

D. generosity

 

Question:- 58-60

In these questions, four alternatives are given for the idiom/ phrase given in bold. Choose the alternative which best expresses the meaning of the idiom/phrase given in bold

 

Q. 58 cut the mustard

A. to get under expectations

B. to score average

C. to perform well

D. to underperform

 

Q. 59 a chip off the old block

A. reminds them of one’s father

B. to remind of one’s son

C. reminds them of one’s son

D. reminds of previous memories

 

Q. 60 to fish in troubled waters

A. to indulge in evil conspiracies

B. to make a profit out of disturbance

C. to aggravate the situation

D. to make the most of bad bargain

 

Question:- 61-62

In these questions, a sentence or a part of sentence is printed in bold. Below are given alternatives to the bold part which may. improve the sentence. Choose the correct alternative. In case no improvement is needed, mark· ‘No improvement’

as your answer.

 

Q. 61 What you have been doing since the workshop last month?

A. have you done

B. you have done

C. have you been doing

D. No improvement

 

Q. 62 Corruption is the most serious problem in India.

A. the more serious

B. very serious

C. serious

D. No improvement

 

Question:- 63-67

In the following questions, fill in the blank(s) with suitable word(s) out of the four alternatives.

 

Q. 63 Sid and Harsh are ___ unable to complete the task .

A. neither

B. either

C. each

D. both

 

Q. 64 The examinee could guess ___ the answer correctly

A. at

B. about

C. through

D. to

 

Q. 65 Be ___ and always look to the comfort of others.

A. considerate

B. cautious·

C. considerable

D. consider

 

Q. 66 As usual, a lot of people were ___ in the king’s darbar.

A. their

B. possess

C. past

D. present

 

Q. 67 In this question, four words are given out of which one is correctly spelt. Find the correctly spelt word.

A. Ommineous

B. Omineous

C. Ominous

D. Omenous

 

Question:- 68-71

In the following questions, some parts of the sentences have errors and some are correct. Find out which part of a sentence has an error. The number of that part is your answer. If a sentence is free from errors, your answer is No error.

 

Q. 68 He ought not (1)/ have done such a (2)/ filthy thing. (3)/ No error (4)

A. 1

B. 2

C. 3

D. 4

 

Q. 69 The reason for (1)/ his failure is because (2)/ he did not work hard. (3)/ No error (4)

A. 1

B. 2

C. 3

D. 4

 

Q. 70 I have reached (1)/ the office before (2)/ the rain started. (3)/ No error (4)

A. 1

B. 2

C. 3

D. 4

 

Q. 71 A large (1)/ consignment of books (2)/ are expected. (3)/ No error (4)

A. 1

B. 2

C. 3

D. 4

 

Question:- 72-100

Every profession or trade, every art and every science has its technical vocabulary, the function of which is partly to designate things or processes which have no names in ordinary English and partly to secure greater exactness in nomenclature. Such special dialects or jargons are necessary in technical discussion of any kind. Being universally understood by the devotees of the particular science or art, they have the precision of a mathematical formula. Besides, they save time, for it is much more economical to name a process than to describe it. Thousands of these technical terms are very properly included in every large dictionary, yet, as a whole, they are rather on the outskirts of the English language than actually within its borders Different occupations, however, differ widely in the character of their special vocabularies. In trades and handicrafts and other vocations like farming and fishing that have occupied great numbers of men from remote times, the technical vocabulary is very old. An average man now uses these in his own vocabulary. The special dialects of law, medicine divinity and philosophy have become familiar to cultivated persons.

 

Q. 72 Special words used in technical discussion

A. may become part of common speech

B. never last long

C. should resemble mathematical formula

D. should be confined to scientific fields

 

Q. 73 The writer of this article is:

A. a scientist

B. a politician

C. a linguist

D. a businessman

 

Q. 74 This passage is primarily concerned with ______

A. various occupations and professions

B. technical terminology

C. scientific undertakings

D. a new language

 

Q. 75 It is. true that ____

A. various occupations and professions often interchange words

B. there is always a nontechnical word that may · be substituted for the technical word.

C. the average man often uses in his own vocabulary what was once technical language not meant for him.

D. everyone is interested in scientific findings

 

Q. 76 Which of the following is known as the Manchester of South India?

A. Kochi

B. Vishakhapatanam

C. Coimbatore

D. Bangalore

 

Q. 77 Who was the flag bearer of india at Rio Olympics 2016?

A. P.V. Sindhu

B. Jwala Gutta

C. Yogeshwar Dutt

D. Abhinav Bindra

 

Q. 78 Which of the following is the first cricketer to score 1000 runs in an innings?

A. Sachin Tendulkar

B. Vinod Kamble

C. Pranav Dhanawade

D. Virat Kohli

 

Q. 79 Which of the following is the largest irrigation plant in India?

A. Buckingham Canal

B. Indira Gandhi Canal

C. Upper Ganges Canal

D. Tajewala Canal

 

Q. 80 The soil of Kerala is rich in which of the following soils?

A. Alluvial Soil

B. Laterite Soil

C. Sandy Soil

D. Loamy Soil

 

Q. 81 Which of the process is known as nitrification?

A. Reaction of Nitrogen Monoxide with oxygen to form nitric acid

B. Reaction of nitrogen dioxide with water to form nitric acid

C. Conversion of ammonia to nitrites

D. Conversion of nitrite to nitric oxide

 

Q. 82 Conversion of nitrite to nitric oxide

A. Australia

B. Canada

C. Africa

D. Russia

 

Q. 83 Which country announces the imposition of a three-month state emergency after failed coup?

A. Turkey

B. Syria

C. Sudan

D. Iran

 

Q. 84 Beighton Cup is related to which of the following sport?

A. Football

B. Hockey

C. Badminton

D. Cricket

 

Q. 85 Sun temple is situated in which of the following states?

A. Odisha

B. Gujarat

C. Karnataka

D. Tamil Nadu

 

Q. 86 The growth of bacteria is measured by

A. hemocytometer

B. spectrophotometer

C. calorimeter

D. auxanometer

 

Q. 87 The sideways erosion which widens the river valley called?

A. Lateral Corrosion

B. Vertical Corrosion

C. Side Corrosion

D. Mean Corrosion

 

Q. 88 The Constitution ____

A. is silent on the President’s re-election to the office

B. allows re-election of a per§on to the President’s post

C. restricts a person to remain President for only two terms.

D. has .been amended to ·a1 .. low a person only one term as President

 

Q. 89 Smooth muscles are likely to be found in

A. muscles of legs

B. muscles of arms

C. stomach

D. heart

 

Q. 90 Synagogue is the place of worship of

A. Zoroastrianism

B. Taoism

C. Judaism

D. Shintoism

 

Q. 91 The civilian airport of highest altitude is in

A. Tibet

B. Nepal

C. India

D. China

 

Q. 92 The branch of biology which deals with extinct organisms is called

A. Palynology

B. Phylogeny

C. Palaeobotany

D. Paleontology

 

Q. 93 The least distance of distinct vision is

A. 35 cm

B. 25 cm

C. 45 cm

D. 15 cm

 

Q. 94 When will demand become a grant?

A. When a demand is proposed

B. After the discussion on demand is over

C. After the demand is granted

D. When the budget session is closed

 

Q. 95 Summer rains in Australia broadly. decreases from

A. east to west

B. west to east

C. north to south

D. south to north

 

Q. 96 The blue revolution is related with

A. Fish production

B. Food grain production

C. Oil seed production

D. Milk production

 

Q. 97 Which is post-harvest folk dance in Assam

A. Ankia Nat

B. Bihu

C. Raut Nacha

D. Namgen

 

Q. 98 The substrate of photo respiration is

A. Fructose

B. Pyruvic acid

C. Glycolate

D. Glucose

 

Q. 99 The UNIX operating system is suitable for

A. Multi user

B. Real-Time Processing

C. Distributed Processing

D. Single ·user

 

Q. 100 Sinkhole is a phenomenon of

A. Plain

B. Desert

C. Tundra

D. Karst

 

 

Answer Sheet 
Question 1 2 3 4 5 6 7 8 9 10
Answer D B B C A C A B C C
Question 11 12 13 14 15 16 17 18 19 20
Answer D C D B C D C A B C
Question 21 22 23 24 25 26 27 28 29 30
Answer B A D D B D A D B C
Question 31 32 33 34 35 36 37 38 39 40
Answer D B D B A C C A B B
Question 41 42 43 44 45 46 47 48 49 50
Answer C B B C A D A C C D
Question 51 52 53 54 55 56 57 58 59 60
Answer D D D D A B B C A B
Question 61 62 63 64 65 66 67 68 69 70
Answer C D D A A D C B B A
Question 71 72 73 74 75 76 77 78 79 80
Answer C A C B C C D C B B
Question 81 82 83 84 85 86 87 88 89 90
Answer C D A B A B A B C C
Question 91 92 93 94 95 96 97 98 99 100
Answer A D B C C A B C A D

SSC CGL Tier-I 01 September 2016 Shift-I Previous Year Paper

SSC CGL Tier 1 2016 1 Sep Shift 1

Q. 1 Select the related word from the given alternatives.

Psychology : Human Being :: Ornithology : ?

A. Birds

B. Volcanoes

C. Insects

D. Reptiles

 

Q. 2 Select the related word/letters/number from the given alternatives.

EGIK : FHJL :: MOQS : ?

A. LNOQ

B. NPRT

C. KMOQ

D. NRPT

 

Q. 3 Select the related number from the given alternatives. 

147 : 741 :: 869 : ?

A. 896

B. 968

C. 689

D. 986

 

Q. 4 For the following questions

Find the odd word from the given alternatives.

A. Herb

B. Flower

C. Tree

D. Shrub

 

Q. 5 For the following questions

Find the odd letters pair from the given alternatives.

A. JKLM

B. NOPQ

C. RSTU

D. VWXZ

 

Q. 6 For the following questions

Find the odd word/letters/number pair from the given alternatives.

A. (85, 136)

B. (34, 85)

C. (102, 153)

D. (63, 162)

 

Q. 7 Arrange the following words as per order in the dictionary

1. Organ

2. Origin

3. Orient

4. Organic

5. Organise

A. 1, 5, 3, 4, 2

B. 1, 5, 4, 2, 3

C. 1, 4, 5, 3, 2

D. 1, 4, 5, 2, 3

 

Q. 8 Which one set of letters when sequentially placed at the gaps in the given letter series shall complete it?

_sr_tr_srs_r_srst_

A. ttssrr

B. tsrtsr

C. strtrs

D. tstttr

 

Q. 9 Find the missing number in the following series:

43, 172, 86, 344, ?

A. 172

B. 258

C. 129

D. 430

 

Q. 10 ‘A’ is the sister of ‘B’. ‘B’ is married to ‘D’. ‘B’ and ‘D’ have a daughter ‘G’. How is ‘G’ related to ‘A’?

A. Sister

B. Daughter

C. Niece

D. Cousin

 

Q. 11 Anil is as much younger to Vivek as he is older to Tarun. If the total of the ages of Vivek and Tarun is 48 years, how old is Anil?

A. 26

B. 33

C. 24

D. 18

 

Q. 12 From the given alternative words, select the word which cannot be formed using the letters of the given word:

CHRONOLOGICAL

A. CALL

B. LOGIC

C. CALICO

D. ANALOGY

 

Q. 13 If HOUSE is written as FQSUC , then how can CHAIR be written in that code?

A. DIBJS

B. SBJID

C. SHBGD

D. AJYKP

 

Q. 14 If P denotes ÷ , Q denotes x, R denotes + and S denotes – then what is the value of 18Q12P4R5S6

A. 64

B. 53

C. 81

D. 24

 

Q. 15 If 4 + 3 = 25, and 8 + 4 = 80, then, 3 + 2 = ?

A. 15

B. 10

C. 13

D. 12

 

Q. 16 Select the missing numbers from the given alternatives

A. 30

B. 35

C. 20

D. 25

 

Q. 17 Ramesh starts his journey by walking 2 kilometres towards North. Then he takes a right turn and walks 1 kilometre. Again takes a right turn and walks 2 kilometre. Now which direction is he facing?

A. East

B. West

C. South

D. North

 

Q. 18 Consider the given statement/s to be true and decide which of the given conclusions/assumptions can definitely be drawn from the given statement.

Statement: Irregularity is a cause for failure in exams. Some regular students fail in the examinations.

Conclusions:

I. All failed students are regular.

II. All successful students are not regular

A. Only conclusion I follows

B. Only conclusion II follows

C. Both conclusion I and conclusion II follow

D. Neither conclusion I nor conclusion II follows

 

Q. 19 How many triangles are there in the given figure?

A. 18

B. 19

C. 20

D. 21

 

Q. 20 Which one of the following figures represents the following relationship:

Birds, Parrots, Bats

A. 1

B. 2

C. 3

D. 4

 

Q. 21 Which answer figure will complete the pattern in the question figure?

A. 1

B. 2

C. 3

D. 4

 

Q. 22 From the given answer figures, select the one in which the question figure is

hidden/embedded

A. 1

B. 2

C. 3

D. 4

 

Q. 23 A piece of paper is folded and cut as shown below in the question figures. From the given answer figures, indicate how it will appear when opened.

A. 1

B. 2

C. 3

D. 4

 

Q. 24 If a mirror is placed on the line AB, then which of the answer figures is the right image of the given figure?

A. 1

B. 2

C. 3

D. 4

 

Q. 25 In the question, a word is represented by only one set of numbers as given in any one of the alternatives. The sets of numbers given in the alternatives are represented by two classes of alphabets as in two matrices given below. The columns and rows of Matrix I are numbered from 0 to 4 and that of Matrix II are numbered from 5 to 9. A letter from these matrices can be represented first by its row and next by its column, e.g., ‘B’ can be represented by 00, 13 etc., and ‘A’ can be represented by 55, 69 etc. Similarly you have to identify the set for the word ‘GIRL’

A. 02, 56, 97, 24

B. 31, 79, 68, 42

C. 23, 97, 77, 11

D. 11, 88, 95, 23

 

Q. 26 The minimum short-term natural hazard is __________ .

A. blizzard

B. earthquake

C. volcanic eruption

D. bolt of lightning

Q. 27 DOTS is a treatment given to patients suffering from _________ .

A. Polio

B. AIDS

C. Hepatitis

D. Tuberculosis

 

Q. 28 Which dynasty was ruling in Vijaynagar empire at the time of the Battle of Talikota?

A. Sangam

B. Aniridu

C. Tuluva

D. Saluva

 

Q. 29 Which of the following Sultans died while playing Polo or Chaugan?

A. Qutb-ud-din Aibak

B. Balban

C. Iltutmish

D. Nasiruddin Muhammad

 

Q. 30 Molten rock below the surface of the earth is called __________ .

A. Basalt

B. Laccolith

C. Lava

D. Magma

 

Q. 31 Magnesium is a constituent metal of __________ .

A. Chlorophyll molecule

B. DNA

C. Mitochondria

D. Ribosomes

 

Q. 32 Which one among the following components is used as an amplifying device?

A. Transformer

B. Diode

C. Capacitor

D. Transistor

 

Q. 33 A __________ is an agreement between the communicating parties on how communication is to proceed.

A. Path

B. SLA

C. Bond

D. Protocol

 

Q. 34 Name the gas used in preparation of bleaching powder

A. Oxygen

B. Hydrogen

C. Nitrogen

D. Chlorine

 

Q. 35 The depletion in Ozone layer is caused by ___________ .

A. Nitrous oxide

B. Carbon dioxide

C. Chlorofluorocarbons

D. Methane

 

Q. 36 Sariska and Ranthambore are the reserves for which of the following

A. Lion

B. Deer

C. Tiger

D. Bear

 

Q. 37 The prestigious ‘Jawaharlal Nehru Award for International Understanding’ is instituted by

A. National Centre for Art and Culture

B. department of Youth and Sports

C. Nehru Memorial Museum

D. Indian Council for Cultural Relations

 

Q. 38 Phosphorus is kept in water because

A. its ignition temperature is very high

B. its ignition temperature is very low

C. its critical temperature is high

D. its critical temperature is low

 

Q. 39 The famous Dilwara temples of Mount Abu are a sacred pilgrimage place for the

A. Buddhists

B. Jains

C. Sikhs

D. Parsis

 

Q. 40 The unit of measurement of noise is

A. Decibel

B. Hertz

C. Amplifier

D. Acoustics

 

Q. 41 The commonly used coolant in refrigerators is

A. Ammonia

B. Nitrogen

C. Freon

D. Oxygen

 

Q. 42 The complete form of ‘IC’ in electronics is

A. Internal circuit

B. Independent circuit

C. Integrated circuit

D. Inbuilt circuit

 

Q.43 Who among the following did Gandhiji regard as his political Guru?

A. Mahadev Desai

B. Dayanand Saraswati

C. Acharya Narendra Dev

D. Gopal Krishna Gokhale

 

Q. 44 Which of the following constitutional Amendment Act, deals with the Elementary Education as a Fundamental Right?

A. 84th Amendment Act

B. 85th Amendment Act

C. 86th Amendment Act

D. 87th Amendment Act

 

Q. 45 The World’s largest island is

A. Greenland

B. Iceland

C. New Guinea

D. Madagascar

 

Q. 46 Sea water is saltier than rain water because

A. Sea animals are salt producing

B. The air around the sea is saltish

C. Rivers wash away salts from earth and pour them into the sea

D. Sea beds have salt producing mines

 

Q. 47 Pannalal Ghosh played which of the following musical instruments?

A. Flute

B. Sitar

C. Violin

D. Santoor

 

Q. 48 Rio Summit is associated with

A. Convention on Biological Diversity

B. Green house gases

C. Ozone depletion

D. Wetlands

 

Q. 49 Polyploidy arises due to change in the

A. number of chromatids

B. structure of genes

C. number of chromosomes

D. structure of chromosomes

 

Q. 50 Which South Korean author in May 2016 won the 2016 Man Booker International Prize for her novel ‘The Vegetarian’?

A. Han Kang

B. Yiyun Li

C. Tami Hoag

D. Taya Zinkin

 

Q. 51 A and B together can finish a work in 30 days. They worked for it for 20 days and then B left the work. The remaining work was done by A alone in 20 days more. In how many days can A alone finish the work?

A. 48 days

B. 50 days

C. 54 days

D. 60 days

 

Q. 52 The centroid of an equilateral triangle ABC is G. If AB is 6 cms, the length of AG is 

A. √3 cm

B. 2√3 cm

C. 3√2 cm

D. 2√2 cm

 

Q. 53 A merchant changed his trade discount from 25% to 15%. This would increase selling price by

A. 3⅓%

B. 6⅙%

C. 13⅓%

D. 16⅓%

 

Q. 54 If 177 is divided into 3 parts in the ratio 1/2 : 2/3: 4/5, then the second part is

A. 75

B. 45

C. 72

D. 60

 

Q. 55 If percentage of profit made,when an article is sold for Rs.78, is twice as when it is sold for Rs.69, the cost price of the article is

A. Rs. 49

B. Rs. 51

C. Rs. 57

D. Rs. 60

 

Q. 56 The ratio between Ram’s age and Rahim’s age is 10:11. What is the age of Rahim in percentage of Ram’s age

A. 109(1/11)%

B. 110%

C. 111(1/9)%

D. 111%

 

Q. 57 Gautam travels 160 kms at 32 kmph and returns at 40 kmph. Then average speed 

A. 72 kmph

B. 71.11 kmph

C. 36 kmph

D. 35.55 kmph

 

Q. 58 If x=3/2, then the value of 27×3-54×2+36x-11 is

A. 11⅜

B. 11⅝

C. 12⅜

D. 12⅝

 

Q. 59 If a+b+c = 6 and ab+bc+ca = 1, then the value of bc(b+c) + ca(c+a) +ab(a+b) +3abc is

A. 33

B. 66

C. 55

D. 23

 

Q. 60 If the angles of a triangle are in the ratio of 2:3:4, then the difference of the measure of greatest angle and smallest angle is

A. 20°

B. 30°

C. 40°

D. 50°

 

Q. 61 In ΔABC, ∠A = 90°, AD ┴ BC and AD = BD = 2 cm. The length of CD

A. 3 cm

B. 3.5 cm

C. 3.2 cm

D. 2 cm

 

Q. 62 If tan 45° = cot θ , then the value of θ , in radians is

A. Π

B. Π/9

C. Π/2

D. Π/12

 

Q. 63 (2^51 + 2^52+2^53+2^4+2^55) is divisible by

A. 23

B. 58

C. 124

D. 127

 

Q. 64 The average of 12 numbers is 9. If each number is multiplied by 2 and added to 3, the average of the new set of numbers is

A. 9

B. 18

C. 21

D. 27

 

Q. 65 Choose the correct option

A. 1

B. 3

C. 0

D. 2

 

Q. 66 Choose the correct option

A. 4/5

B. 3/5

C. 8/5

D. 1/5

 

Q. 67 The perimeter of two similar triangles ABC and PQR are 36 cms and 24 cms respectively. If PQ = 10 cm then the length of AB is

A. 18 cm

B. 12 cm

C. 15 cm

D. 30 cm

 

Q. 68 n a triangle ABC, AB = 8 cm, AC = 10 cm and ∠B = 90°, then the area of ΔABC is

A. 49 sq.cm

B. 36 sq.cm

C. 25 sq.cm

D. 24 sq.cm

 

Q. 69 Choose the correct option

A. 1/√2

B. 0

C. 1/2

D. √3/2

 

Q. 70 The compound interest on Rs. 64,000 for 3 years, compounded annually at 7.5% p.a. is

A. Rs. 14,400

B. Rs. 15,705

C. Rs. 15,507

D. Rs. 15,075

 

Q. 71 The angles of elevation of the top of a temple, from the foot and the top of a building 30 m high, are 60° and 30° respectively. Then height of the temple is

A. 50 m

B. 43 m

C. 40 m

D. 45 m

 

Question:- 72-75

Study the pie-chart given and answer the following questions.

 

Q. 72 If the miscellaneous charges are Rs. 6000, then the advertisement charges are

A. Rs.12000

B. Rs.27000

C. Rs.90000

D. Rs.25000

 

Q. 73 The central angle of printing charge is x more than that of advertisement charge. Then the value of x is

A. 72°

B. 61.2°

C. 60°

D. 54.8°

 

Q. 74 What should be the central angle of the sector ‘cost of paper’?

A. 22.5°

B. 54.8°

C. 36°

D. 16°

 

Q. 75 The ratio between royalty and binder’s charges is

A. 5:6

B. 5:8

C. 6:5

D. 8:13

 

Q. 76 In the following question, out of the four alternatives, choose the word which best expresses the meaning of the given word and click the button corresponding to it.

RECEPTACLE

A. COMPARTMENT

B. HOLE

C. CONTAINER

D. FUNNEL

 

Q. 77 In the following question, out of the four alternatives, choose the word which is opposite in meaning to the given word and click the button corresponding to it.

VANITY

A. PRIDE

B. LOVE

C. COURAGE

D. HUMILITY

 

Q. 78 Four words are given, out of which only one word is spelt correctly. Choose the correctly spelt word and click the button corresponding to it.

A. Acquisision

B. Acqiusition

C. Acquisition

D. Acquisetion

 

Q. 79 In the following questions, one part of the sentence may have an error. Find out which part of the sentence has an error and click the button corresponding to it. If the sentence is free from error, click the “No error” option.

If you permit me to speak the truth (A) / I shall state without hesitation (B) / that you have done a mistake (C) / No Error (D)

A. A

B. B

C. C

D. D

 

Q. 80 In the following questions, one part of the sentence may have an error. Find out which part of the sentence has an error and click the button corresponding to it. If the sentence is free from error, click the “No error” option.

I succeeded persuading him (A) / to come with me (B) / only after hours of argument (C) / No Error (D)

A. A

B. B

C. C

D. D

 

Q. 81 In the following questions, one part of the sentence may have an error. Find out which part of the sentence has an error and click the button corresponding to it. If the sentence is free from error, click the “No error” option.

Vermin (A) / does much harm (B) / to crops (C) / No Error (D)

A. A

B. B

C. C

D. D

 

Q. 82 The sentences given with blanks are to be filled with an appropriate word(s). Four alternatives are suggested for each question. For each question, choose the correct alternative and click the button corresponding to it.

Light is _____ than sound.

A. faster

B. more faster

C. sounder

D. lighter

 

Q. 83 The sentences given with blanks are to be filled with an appropriate word(s). Four alternatives are suggested for each question. For each question, choose the correct alternative and click the button corresponding to it.

The man _____ for the purpose never did what was expected of him.

A. supplied

B. favoured

C. employed

D. cited

 

Q. 84 The sentences given with blanks are to be filled with an appropriate word(s). Four alternatives are suggested for each question. For each question, choose the correct alternative and click the button corresponding to it.

There was a serious _____ between the two brothers.

A. altercation

B. alteration

C. aberration

D. altieration

 

Q. 85 In each of the questions, four alternatives are given for the Idiom/Phrase. Choose the alternative which best expresses the meaning of the Idiom/Phrase and click the button corresponding to it.

To give oneself airs

A. Space to breathe

B. Time

C. Acting strange

D. Behaving arrogantly

 

Q. 86 In each of the questions, four alternatives are given for the Idiom/Phrase. Choose the alternative which best expresses the meaning of the Idiom/Phrase and click the button corresponding to it.

At a stone’s throw

A. At a short distance

B. At a place where quarrels take place

C. At a great distance

D. A quarry

 

Q. 87 In each of the questions, four alternatives are given for the Idiom/Phrase. Choose the alternative which best expresses the meaning of the Idiom/Phrase and click the button corresponding to it.

Bone of contention

A. A delicious non-vegetarian item

B. An item which made them content

C. Cause for quarrel

D. A link between them

 

Q. 88 Out of the four alternatives, choose the one which can be substituted for the given words/sentences and click the button corresponding to it.

A person who has lost the protection of the law

A. Outlaw

B. Immigrant

C. Outcast

D. Orphan

 

Q. 89 Out of the four alternatives, choose the one which can be substituted for the given words/sentences and click the button corresponding to it.

Falsification of documents etc

A. Xeroxing

B. Forgery

C. Laminating

D. Copying

 

Q. 90 Out of the four alternatives, choose the one which can be substituted for the given words/sentences and click the button corresponding to it.

To make atonement for one’s sins

A. Expiate

B. Renounce

C. Remonstrate

D. Recant

 

Q. 91 A sentence/a part of the sentence is under double quotes. Four alternatives are given to the underlined part which will improve the sentence. Choose the correct alternative and click the button corresponding to it. In case no improvement is needed, click the button corresponding to “No improvement”.

The boys “absented themselves” for two days.

A. were absented

B. absented

C. had absented

D. No improvement

 

Q. 92 A sentence/a part of the sentence is under double quotes. Four alternatives are given to the underlined part which will improve the sentence. Choose the correct alternative and click the button corresponding to it. In case no improvement is needed, click the button corresponding to “No improvement”.

My mother insists on setting “up” something everyday for charity.

A. on

B. in

C. aside

D. No improvement

 

Q. 93 A sentence/a part of the sentence is under double quotes. Four alternatives are given to the underlined part which will improve the sentence. Choose the correct alternative and click the button corresponding to it. In case no improvement is needed, click the button corresponding to “No improvement”.

He “began” on a new business venture.

A. embarked

B. begun

C. opened

D. No improvement

 

Q. 94 A sentence/a part of the sentence is under double quotes. Four alternatives are given to the underlined part which will improve the sentence. Choose the correct alternative and click the button corresponding to it. In case no improvement is needed, click the button corresponding to “No improvement”.

I have to “say” them the truth

A. tell

B. state

C. speak

D. No improvement

 

Q. 95 A sentence/a part of the sentence is under double quotes. Four alternatives are given to the underlined part which will improve the sentence. Choose the correct alternative and click the button corresponding to it. In case no improvement is needed, click the button corresponding to “No improvement”.

The SSB will interview the candidate between “11 a.m. and 3.00 p.m.”

A. 11.00 am to 3 pm

B. 11.00 from 3 pm

C. 11.am by 3.00 pm

D. No improvement

 

Questions: 96 – 100 

A passage is given with 5 questions following it. Read the passage carefully and choose the best answer to each question out of the four alternatives and click the button corresponding to it. 

Settled life and cultivation gave man leisure; he had no longer to always think of getting food. During spare time he could make stone tools, hoes or pots and weave cloth. Some people spared from producing their own food could even devote themselves to other activities all the time. This resulted in a division of labour. The division of labour made it possible for various groups to specialize, that is, to acquire greater skill and learn better techniques in doing one kind of work. The settled community life needed rules to regulate the behaviour of the members of the community. It is not possible to know exactly how regulations were established. It appears that the decisions regarding the community were taken by the people as a whole, or by a council of elders, as is in the practice in tribal societies. There were perhaps no kings or any organized government. Most likely, there were chiefs elected by the community for their qualities of leadership. But, these chiefs could not pass their positions on to their sons and they enjoyed few special privileges. Archaeological excavations have not revealed anything which would indicate the prevalence of a higher status for some members of the community. This is also supported by the study of life in many tribes in modern times. Thus, social inequalities do not seem to have emerged even in Neolithic times.

 

Q. 96 What did man do in his spare time?

A. Made stone tools

B. Made hoes or pots

C. Weave cloth

D. All of the above

 

Q. 97 In the passage, “division of labour” means?

A. Specialization to acquire greater skills

B. Learning arithmetic

C. Working in groups

D. Working from home

 

Q. 98 To which period do the Neolithic times refer:

A. Later part of the Stone Age

B. Early part of the Stone Age

C. Later part of the Ice Age

D. Medieval times

 

Q. 99 Chiefs were elected by the community for their

A. age

B. qualities of leadership

C. responsibility and courage 

D. trust and loyalty

 

Q. 100 The meaning of ‘privilege’ is

A. allowance

B. favour

C. benefit

D. interest

 

 

Answer Sheet 
Question 1 2 3 4 5 6 7 8 9 10
Answer A D B B D D C D A C
Question 11 12 13 14 15 16 17 18 19 20
Answer C D D B C B C D A C
Question 21 22 23 24 25 26 27 28 29 30
Answer B C D A C D D C A D
Question 31 32 33 34 35 36 37 38 39 40
Answer A D D D C C D B B A
Question 41 42 43 44 45 46 47 48 49 50
Answer C C D C A C A A C A
Question 51 52 53 54 55 56 57 58 59 60
Answer D B C D D B D D B C
Question 61 62 63 64 65 66 67 68 69 70
Answer D C C C C C C D C C
Question 71 72 73 74 75 76 77 78 79 80
Answer D B B C A C D C C A
Question 81 82 83 84 85 86 87 88 89 90
Answer B A C A D A C A B A
Question 91 92 93 94 95 96 97 98 99 100
Answer D C A A D D A A B C

SSC CGL Tier-I 19 August 2014 Shift-I Previous Year Paper

SSC CGL Tier 1 2014 19 Aug shift 1 

Q. 1 Voyage : Sea sickness : : Heights : ?

A. Ship
B. Travel
C. Giddiness
D. Motion

Q. 2 Waitress : Restaurant : : ?

A. Doctor : Nurse
B. Driver : Truck
C. Teacher : School
D. Actor : Role

Q. 3 AROUND : RAUODN : : GROUND : ?

A. RGUODN
B. NDOOGR
C. OUNDGR
D. DNUURG

Q. 4 APPROACHED : ROACHEDAPP : : BARGAINED : ?

A. AINEDBARG
B. GAINEDBAR
C. GAINEDRAB
D. RABGAINED

Q. 5 8 : 256 : : ?

A. 7 : 343
B. 9 : 243
C. 10 : 500
D. 5 : 75

Q. 6 21 : 3 : : 574 : ?

A. 23
B. 82
C. 97
D. 113

Q. 7 Choose the correct option:

A. Obstetrician
B. Podiatrist
C. Pulmonologist
D. Prosthetist

Q. 8 Choose the correct option:

A. Century
B. Decadent
C. Year
D. Month

Q. 9 Choose the correct option:

A. FEDC
B. STUV
C. LKJI
D. RQPO

Q. 10 Choose the correct option:

A. LPXOY
B. RQST
C. FBDLX
D. MPONL

Q. 11 Choose the correct option:

A. 14 – 16
B. 56 – 64
C. 77 – 88
D. 80 – 93

Q. 12 Choose the correct option:

A. 13 – 21
B. 19 – 27
C. 15 – 23
D. 16 – 24

Q. 13 Which one of the given responses would be a meaningful order of the following ?
1. apartment 2. town 3. street 4. building 5. complex

A. 1, 5, 4, 3, 2
B. 4, 5, 3, 2, 1
C. 2, 1, 3, 4, 5
D. 1, 4, 5, 3, 2

Q. 14 If the following words are arranged in reverse dictionary order, which word comes second ?

A. Explosion
B. Express
C. Exploit
D. Expulse

Q. 15 Which one set of letters when sequentially placed at the gaps in the given letter series shall complete it ?
ac__c_cb_acbcacbca_bc

A. abbb
B. bacc
C. babc
D. bbcc

Q. 16 AZ, CX, FU, ?

A. IR
B. IV
C. JQ
D. KP

Q. 17 1, 2, 6, 24, ? ,720

A. 3
B. 5
C. 120
D. 8

Q. 18 156, 506, ?, 1806

A. 1056
B. 856
C. 1456
D. 1506

Q. 19 Suket has three daughters and each daughter has a brother. How many male members are there in the family ?

A. 4
B. 3
C. 2
D. 1

Q. 20 Dinesh and Ramesh start together from a certain point in the opposite direction on motorcycles. The speed of Dinesh is 60 km per hour and Ramesh 44 km per hour. What will be the distance between them after 15 minutes ?

A. 20 km
B. 24 km
C. 26 km
D. 30 km

Q. 21 An insect is walking in a straight line. It covers a distance of 15 cm per minute. It comes back 2.5 cm after every 15 cm. How long will it take to cover a distance of 1 metre ?

A. 6.5 min
B. 8 min
C. 10 min
D. 12 min

Q. 22 A group of alphabets are given with each being assigned a number. These have to be unscrambled into a meaningful word and correct order of letters may be indicated from the given responses.
T M H R E O
5 4 3 2 1 0

A. 025314
B. 315402
C. 405312
D. 504231

Q. 23 From the given alternative words, select the word which cannot be formed using the letters of the given word :
TRIVANDRUM

A. RAIN
B. DRUM
C. TRAIN
D. DRUK

Q. 24 How many meaningful English words can be made with the letters ‘ OEHM ‘ using each letter only once in each word ?

A. FOUR
B. THREE
C. TWO
D. ONE

Q. 25 A shopkeeper quotes the rate on the price tag by replacing numbers with letters as follows :
0 1 2 3 4 5 6 7 8 9
B R O W N S T I C K
If a customer purchases two items whose price tags read ₹IIT and ₹NICK, what is the total amount he has to Pay?

A. ₹ 4776
B. ₹ 4765
C. ₹ 5565
D. ₹ 5665

Q. 26 If CASUAL is coded as SACLAU, then what would be the code of MATRIC ?

A. CIRTAM
B. TMAICR
C. TAMCIR
D. ATMCIR

Q. 27 If ‘S’ is written as ‘H’ ‘R’ as ‘@’ ‘A’ as ‘∇’ ‘M’ as ‘#’ , ‘T’ as ‘$’ and ‘E’ as ‘%’ then how is ‘MASTER’ written in that code ?

A. #∇H$%@
B. #H∇$%@
C. #∇$H%@
D. #∇H%@$

Q. 28 If 1 candle in box number 1 is placed in box number 2, then box-2 has twice the number of candles that box 1 has. If 1 candle from box-2 is placed in box-1, the box-2 and box-1 have the same number of candles. How many candles were there in box-1 and box-2 ?

A. 5 : 3
B. 7 : 5
C. 6 : 4
D. 5 : 7

Q. 29 Which of the following interchange of signs would make the equation correct ?
6 × 4 + 2 = 16

A. + and ×, 2 & 4
B. + and ×, 4 & 6
C. + and ×, 2 & 6
D. + and ×, 3 & 4

Q. 30 Select the correct combination of mathematical sings to replace the * signs and to balance the following equation :
45 * 3 * 6 * 2 * 54

A. + × ÷ =
B. + ÷ × =
C. + × – =
D. + + – =

Q. 31 Select the correct combination of mathematical signs to replace * signs and to balance the following equation :
8 * 5 * 10 * 2 * 25

A. + × ÷ =
B. + ÷ – =
C. × + = ×
D. × – = ×

Q. 32 

7 6 6
8 6 ?
3 4 5
168 144 120

A. 8
B. 10
C. 5
D. 4

Q. 33 

8 5 6
3 7 5
1 4 2
74 90 ?

A. 65
B. 85
C. 52
D. 76

Q. 34
22 46 24
27 58 31
32 68 ?

A. 46
B. 36
C. 32

Q. 35 On one side of a street are even numbers and on the other side are odd numbers. No. 1 is exactly in front of No. 2. My House is No. 9. From my house , a man comes up from No. 2 and knocks at the door, five doors beyond the house in front of me. What is the No. of that house ?

A. 18
B. 20
C. 22
D. 26

Q. 36 Four players P, Q, R and S are standing a play field in such a way that Q is to East of P, R is to the South of P and S is to the North of P. In which direction of Q is S Standing ?


A. North
B. South
C. North-West
D. South-East

Q. 37 Statement : The rich must live more simply. All poor people are simple.
Conclusions :
I. Rich people waste money.
II. Poor people save money.

A. Only I follows
B. Only II follows
C. Neither I nor II follow
D. Both I and II follow

Q. 38 

(1) All roses in Sita’s garden are red.
(2) All marigold flowers in Sita’s garden are orange.
(3) All flowers in Sita’s garden are either red or orange.
If 1st two statements are true, the third is

A. True
B. False
C. Uncertain
D. Vague

Q. 39 Find out which answer figure will exactly make up the question figure.

A. a
B. b
C. c
D. d

Q. 40 How many triangles are there in the given figure ?

A. 48
B. 60
C. 56
D. 52

Q. 41 Choose the cube that will be formed by folding the sheet of paper shown in the problem figure.

A. a
B. b
C. c
D. d

Q. 42 In a group of persons, 11 persons speak Kannada, 20 persons speak Tamil and 11 persons speak Telugu. In that group, if two persons speak two languages and one person speak all the languages, then how many persons are there in the group ?

A. 40
B. 41
C. 42
D. 43


Q. 43 Which one of the following diagrams best depicts the relationship among Human Society – Youth Club, Political Party and Youths ?

A. a
B. b
C. c
D. d

Q. 44 Which one of the following diagrams represent correct relationship among pigeon, birds, dogs ?

A. 1
B. 2
C. 3
D. 4

Q. 45 Which answer figure will complete the pattern in the question figure ?

A. a
B. b
C. c
D. d

Q. 46 Which answer figure will complete the pattern in the question figure ?

A. 1
B. 2
C. 3
D. 4

Q. 47 From the given answer figures, select the figure which is hidden/embedded in the question figure.

A. a
B. b
C. c
D. d

Q. 48 A piece of paper is folded and cut as shown below in the question figures. From the given answer figures, indicate how it will appear when opened.

A. a
B. b
C. c
D. d

Q. 49 If a mirror is place on the line MN, then which of the answer figures is the right image of the given figure ?

A. a
B. b
C. c
D. d

Q. 50 A word is represented by only one set of numbers as given in any one of the alternatives. The sets of numbers given in the alternatives are represented by two classes of alphabets as in two matrices given below. The columns and rows of Matrix I are numbered from 0 to 4 and that of Matrix II are numbered from 5 to 9. A letter from these matrices can be represented first by its row and next by its column, e.g., ‘F’ can be represented by 30, 22, etc. and ‘N’ can be represented by 97, 89, etc. Similarly, you have to identify the set for the given word.
“DAKU”

A. 95, 40, 04, 42
B. 24, 95, 20, 27
C. 88, 24, 10, 34
D. 57, 13, 23, 21

Q. 51 Prof. Milton Friedman was the leader of

A. Ohio school
B. Chicago school
C. Cambridge school
D. London school

Q. 52 Which one of the following is not a qualitative control of credit by the Central Bank of a country?

A. Rationing of credit
B. Regulation of consumer credit
C. Variation of the reserve ratio
D. Regulation of margin requirements

Q. 53 The market in which loans of money can be obtained is called

A. Reserve market
B. Institutional market
C. Money market
D. Exchange market

Q. 54 If the marginal return increases at a diminishing rate, the total return

A. increases
B. decreases
C. remains constant
D. becomes zero

Q. 55 The law of Increasing Returns means

A. increasing cost
B. decreasing cost
C. increasing production
D. increasing income

Q. 56 The most important feature of Cabinet system of Government is

A. Individual responsibility
B. Collective responsibility
C. Responsibility to none
D. Non-responsibility

Q. 57 Direct legislation in Switzerland has

A. a natural growth
B. a haphazard growth
C. an artificial growth
D. None of the above

Q. 58 Who gave the idea of “Cabinet Dictatorship”?

A. Muir
B. Lowell
C. Marriot
D. Laski

Q. 59 In which of the following countries are the judges of the federal court elected by the two Houses of the Federal Legislature?

A. Switzerland
B. Germany
C. Canada
D. Both (a) and (b)

Q. 60 The President of the USA appoints Supreme Court Judges

A. with Senate’s consent
B. at his discretion
C. with consent of the House of Representatives
D. None of these

Q. 61 Multan was named by the Arabs as

A. City of beauty
B. City of wealth
C. City of gold
D. Pink city

Q. 62 Which one of the following was the book written by Amoghvarsha, the Rashtrakuta King?

A. Adipurana
B. Ganitasara Samgraha
C. Saktayana
D. Kavirajamarga

Q. 63 Who built the Kailasanatha Temple at Ellora?

A. Rajendra I
B. Mahendra Varman I
C. Krishna I
D. Govinda I

Q. 64 The land measures of the Second Pandyan Empire was mentioned in

A. Thalavaipuram Copper Plates
B. Uttirameru Inscription
C. Kudumiyammalai Inscription
D. Kasakudi Copper Plates

Q. 65 Who was the greatest ruler of the Satavahanas?

A. Satkarni I
B. Gautamiputra Satkarni
C. Simuka
D. Hala

Q. 66 Cactus is referred to as


A. Hydrophyte
B. Mesophyte
C. Xerophyte
D. Epiphyte

Q. 67 Which of the following is not a renewable resource?


A. Thorium
B. Geothermal heat
C. Tidal power
D. Radiant energy

Q. 68 Which of the following statements is correct?

A. Mahadeo hills are in the west of Maikala hills
B. Mahadeo hills are the part of Karnataka Plateau
C. Mahadeo hills are in the east of Chotanagpur Plateau
D. Mahadeo hills are the part of Aravalli ranges

Q. 69 Which one of the following pairs is not correctly matched?

A. Hevea Tree—Brazil
B. Sumatra Storm—Malaysia
C. Kajan River—Borneo
D. Dekke Toba fish—Brazil

Q. 70 Which of the following resources is renewable one?

A. Uranium
B. Coal
C. Timber
D. Natural Gas

Q. 71 How many neck canal cells are found in the archegonium of a fern?

A. One
B. Two
C. Three
D. Four

Q. 72 Which angiosperm is vesselless?

A. Hydrilla
B. Trochodendron
C. Maize
D. Wheat

Q. 73 Who was the first child born after operative procedure?

A. Caesar
B. Huxley
C. William
D. Pasteur

Q. 74 Myrmecology is study of

A. Insects
B. Ants
C. Crustaceans
D. Arthropods

Q. 75 NIN (National Institute of Nutrition) Central Office is located at

A. Hyderabad
B. Mumbai
C. Bengaluru
D. Kolkata

Q. 76 HIV often changes is shapes due to the presence of an enzyme called

A. Reverse Transcriptase
B. Enterokinase
C. Nucleotidase
D. Nucleoditase

Q. 77 Fleming’s right hand rule is used to find the direction of the

A. Alternate current
B. Direct current
C. Induced current
D. Actual current

Q. 78 The unit of electrical power is

A. Volt
B. Watt
C. Kilowatt hour
D. Ampere

Q. 79 The resistance of the human body (dry condition) is of the order of

A. 10¹ ohm
B. 10² ohm
C. 10³ ohm
D. 10⁴ ohm

Q. 80 Certain substances lose their electrical resistance completely at super low temperature. Such substances are called

A. super conductors
B. semi conductors
C. dielectrics
D. perfect conductors

Q. 81 The section of the CPU that selects, interprets and monitors the execution of program instructions is

A. Memory
B. Register unit
C. Control unit
D. ALU

 

Q. 82 Who among the following introduced the world’s first laptop computer in the market?

A. Hewlett-Packard
B. Epson
C. Laplink travelling software Inc
D. Microsoft

Q. 83 Brass contains

A. Copper and Zinc
B. Copper and Tin
C. Copper and Silver
D. Copper and Nickel

Q. 84 Which is the purest commercial form of iron?

A. Pig iron
B. Steel
C. Stainless steel
D. Wrought iron

Q. 85 In galvanization, iron is coated with

A. Copper
B. Zinc
C. Tin
D. Nickel

Q. 86 Which one of the following is also known as solution?

A. A compound
B. A homogeneous mixture
C. A heterogeneous mixture
D. A suspension

Q. 87 The cells which are closely associated and interacting with guard cells are

A. Transfusion tissue
B. Complementary cells
C. Subsidiary cells
D. Hypodermal cells

Q. 88 Conversion of starch to sugar is essential for

A. Stomatal opening
B. Stomatal closing
C. Stomatal formation
D. Stomatal growth

Q. 89 Soil erosion can be prevented by

A. Increasing bird population
B. Afforestation
C. Removal of vegetation
D. Overgrazing

Q. 90 Natural sources of air pollution are

A. Forest fires
B. Volcanic eruptions
C. Dust storm
D. Smoke from burning dry leaves

Q. 91 Which of the following Genetically Modified vegetable is recently being made available in Indian market?

A. Carrot
B. Radish
C. Brinjal
D. Potato

Q. 92 “Bull’s eye” is used in the game of

A. Boxing
B. Basketball
C. Polo
D. Shooting

Q. 93 As per newspapers report what percent of Government stake will be disinvested in Rashtriya Ispat Nigam Ltd. (RINL)?

A. 5%
B. 50%
C. 10%
D. 12%

Q. 94 Pablo Picasso, the famous painter was

A. French
B. Italian
C. Flemish
D. Spanish

Q. 95 Which of the following is the Regulator of the credit rating agencies in India?

A. RBI
B. SBI
C. SIDBI
D. SEBI

Q. 96 Which is the first Indian Company to be listed in NASDAQ?

A. Reliance
B. TCS
C. HCL
D. Infosys

Q. 97 RRBs are owned by

A. Central Government
B. State Government
C. Sponsor Bank
D. Jointly by all of the above

Q. 98 The Monetary and Credit Policy is announced by which of the following?

A. Ministry of Finance of Centre
B. Reserve Bank of India
C. State Bank of India
D. Planning Commission of India

Q. 99 Which of the following method is not used in determining National Income of a country?

A. Income Method
B. Output Method
C. Input Method
D. Investment Method

Q. 100 What does the letter ‘e’ denotes in the term ‘e-banking’?

A. Essential Banking
B. Economic Banking
C. Electronic Banking
D. Expansion Banking

Q. 101 Arrange the following in ascending order
3³⁴, 2⁵¹, 7¹⁷, we get

A. 3³⁴ > 2⁵¹ > 7¹⁷
B. 7¹⁷ > 2⁵¹ > 3³⁴
C. 3³⁴ > 7¹⁷ > 2⁵¹
D. 2⁵¹ > 3³⁴ > 7¹⁷

Q. 102 If the product of first fifty positive consecutive integers be divisible by 7^n, where n is an integer, then the largest possible value of n is

A. 7
B. 8
C. 10
D. 5

Q. 103 A, B and C together can do a piece of work in 40 days. After working with B and C for 16 days, A leaves and then B and C complete the remaining work in 40 days more. A alone could do the work in

A. 80 days
B. 90 days
C. 100 days
D. 120 days

Q. 104 Three pipes A, B and C can fill a tank in 6 hours. After working it together for 2 hours, C is closed and A and B can fill the remaining part in 7 hours. The number of hours taken by C alone to fill the tank is

A. 10
B. 12
C. 14
D. 16

Q. 105 Pratibha is thrice as efficient as Sonia and is therefore able to finish a piece of work in 60 days less than Sonia. Pratibha and Sonia can individually complete the work respectively in 

A. 30, 60 days
B. 60, 90 days
C. 30, 90 days
D. 40, 120 days

Q. 106 The base of a right pyramid is an equilateral triangle of side 4 cm each. Each slant edge is 5 cm long. The volume of the pyramid is

A. 4√8/3 cm³
B. 4√60/3 cm³
C. 4√59/3 cm³
D. 4√61/3 cm³

Q. 107 There are two cones. The curved surface area of one is twice that of the other. The slant height of the latter is twice that of the former. The ratio of their radii is

A. 4 : 1
B. 4 : 3
C. 3 : 4
D. 1 : 4

Q. 108 A wire is bent into the form of a circle, whose area is 154 cm². If the same wire is bent into the form of an equilateral triangle, the approximate area of the equilateral triangle is

A. 93.14 cm²
B. 90.14 cm²
C. 83.14 cm²
D. 39.14 cm²

Q. 109 A shopkeeper allows 10% discount on goods when he sells without credit. Cost price of his goods is 80% of his selling price. If he sells his goods by cash, then his profit is

A. 50%
B. 70%
C. 25%
D. 40%

Q. 110 A dealer of scientific instruments allows 20% discount on the marked price of the instruments and still makes a profit of 25%. If his gain over the sale of an instrument is ₹150, find the marked price of the instrument

A. ₹ 938.50
B. ₹ 940
C. ₹ 938
D. ₹ 937.50

Q. 111 Ram bought a T.V. with 20% discount on the labelled price. Had he bought it with 30% discount he would have saved ₹ 800. The value of the T.V. set that he bought is

A. ₹ 5,000
B. ₹ 8,000
C. ₹ 9,000
D. ₹ 10,000

Q. 112 A vessel full of pure acid contains 10 litres of it, of which 2 litres are withdrawn. The vessel is then filled with water. Next 2 litres of the mixture are withdrawn, and again the vessel is filled up with water. The ratio of the acid left in the vessel with that of the original quantity is

A. 1 : 5
B. 4 : 5
C. 4 : 25
D. 16 : 25

Q. 113 Gold is 19 times as heavy as water and copper is 9 times as heavy as water. In what ratio should these be mixed to get an alloy 15 times as heavy as water?

A. 1 : 1
B. 1 : 2
C. 2 : 3
D. 3 : 2

Q. 114 The average age of boys in the class is twice the number of girls in the class. The ratio of boys and girls in the class of 50 is 4 : 1. The total of the ages (in years) of the boys in the class is

A. 2000
B. 2500
C. 800
D. 400

Q. 115 There are 100 students in 3 sections A, B and C of a class. The average marks of all the 3 sections was 84. The average of B and C was 87.5 and the average marks of A is 70. The number of students in A was

A. 30
B. 35
C. 20
D. 25

Q. 116 A sold an article to B at 20% profit and B sold it to C at 15% loss. If A sold it to C at the selling price of B, then A would make

A. 5% profit
B. 2% profit
C. 2% profit
D. 5% loss

Q. 117 The monthly salaries of A and B together amount to ₹40,000. A spends 85% of his salary and B, 95% of his salary. If now their savings are the same, then the salary (in ₹) of A is

A. 10,000
B. 12,000
C. 16,000
D. 18,000

Q. 118 It takes 8 hours for a 600 km journey, if 120 km is done by train and the rest by car. It takes 20 minutes more if 200 km is down by train and the rest by car. The ratio of the speed of the train to that of the car is

A. 2 : 3
B. 3 : 2
C. 3 : 4
D. 4 : 3

Q. 119 If a train runs at 70 km/hour, it reaches its destination late by 12 minutes. But if it runs at 80 km/hour, it is late by 3 minutes. The correct time to cover the journey is

A. 58 minutes
B. 2 hours
C. 1 hour
D. 59 minutes

Q. 120 A man borrowed some money from a private organisation at 5% simple interest per annum. He lended 50% of this money to another person at 10% compound interest per annum and thereby the man made a profit of ` 13,205 in 4 years. The man borrowed

A. ₹ 80,000
B. ₹ 1,00,000
C. ₹ 1,20,000
D. ₹ 1,50,000

Q. 121 If a² + b² + c² = 2a – 2b – 2, then the value of 3a – 2b + c is

A. 0
B. 3
C. 5
D. 2

Q. 122 If a + b + c = 3, a²+ b² + c² = 6 and 1/a + 1/b + 1/c = 1, where a, b, c are all non-zero, then ‘abc’ is equal to

A. 2/3
B. 3/2
C. 1/2
D. 1/3

Q. 123 If a² – 4a – 1 = 0, a ≠ 0, then the value of a² + 3a + 1/a² – 3/a is

A. 24
B. 26
C. 28
D. 30

Q. 124 The total area (in sq. unit) of the triangles formed by the graph of 4x + 5y = 40, x-axis, y-axis and x = 5 and y = 4 is

A. 10
B. 20
C. 30
D. 40

Q. 125 For what value of k, the system of equations kx + 2y = 2 and 3x + y = 1 will be coincident?

A. 2
B. 3
C. 5
D. 6

Q. 126 If x = 2 + √3 , then x² + 1/x² is equal to

A. 10
B. 12
C. -12
D. 14

Q. 127 If a = 4.965, b = 2.343 and c = 2.622, then the value of a³ – b³ – c³ – 3abc is

A. -2
B. -1
C. 0
D. 9.93²

Q. 128 If x + y + z = 0, then the value of x² + y² + z²/(x² – yz) is

A. -1
B. 0
C. 1
D. 2

Q. 129 The value of sin²1° + sin²2° + sin²3° + …. + sin²89° is

A. 22
B. 44
C. 22 1/2
D. 44 1/2

Q. 130 The value of (cos³θ + sin³θ/(cos θ + sin θ)) + (cos³θ – sin³θ/(cos θ – sin θ)) is equal to

A. -1
B. 1
C. 2
D. 0

Q. 131 The shadow of a tower standing on a level plane is found to be 30 m longer when the Sun’s altitude changes from 60° to 45°. The height of the tower is

A. 15(3 + √3) m
B. 15(3 + √1) m
C. 15(3 – √1) m
D. 15(3 – √3) m

Q. 132 If sin 17° = x/y then sec 17° – sin 73° is equal to

A. a
B. b
C. c
D. d

Q. 133 If θ is a positive acute angle and cosec θ + cot θ = √3 , then the value of cosec θ is

A. 1/√3
B. √3
C. 2/√3
D. 1

Q. 134 If cos α + sec α = √3 , then the value of cos³ α + sec³ α is

A. 2
B. 1
C. 0
D. 4

Q. 135 If sin θ + cos θ = √2 cos θ, then the value of cot θ is

A. √2 + 1
B. √2 – 1
C. √3 – 1
D. √3 + 1

Q. 136 In a quadrilateral ABCD, the bisectors of ∠A and ∠B meet at O. If ∠C = 70° and ∠D = 130°, then measure of ∠AOB is

A. 40°
B. 60°
C. 80°
D. 100°

Q. 137 In ΔABC, E and D are points on sides AB and AC respectively such that ∠ABC = ∠ADE. If AE = 3 cm, AD = 2 cm and EB = 2 cm, then length of DC is

A. 4 cm
B. 4.5 cm
C. 5.0 cm
D. 5.5 cm

Q. 138 In a circle with centre O, AB is a chord, and AP is a tangent to the circle. If ∠AOB = 140°, then the measure of ∠PAB is

A. 35°
B. 55°
C. 70°
D. 75°

Q. 139 In DABC, ∠A < ∠B. The altitude to the base divides vertex angle C into two parts C1 and C2, with C2 adjacent to BC. Then

A. C1 + C2 = A + B
B. C1 – C2 = A – B
C. C1 – C2 = B – A
D. C1 + C2 = B -A

Q. 140 If O is the in-centre of ΔABC; if ∠BOC = 120°, then the measure of ∠BAC is

A. 30°
B. 60°
C. 150°
D. 75°

Q. 141 Two parallel chords of a circle of diameter 20 cm are 12 cm and 16 cm long. If the chords are in the same side of the centre, then the distance between them is

A. 28 cm
B. 2 cm
C. 4 cm
D. 8 cm

Q. 142 The interior angle of a regular polygon is 140°. The number of sides of that polygon is

A. 9
B. 8
C. 7
D. 6

Q. 143 If two circles of radii 9 cm and 4 cm touch externally, then the length of a common tangent is

A. 5 cm
B. 7 cm
C. 8 cm
D. 12 cm

Q. 144 The ratio of the total number of students scoring marks less than 50% to that of scoring marks exactly 50% is

A. 50 : 3
B. 25 : 2
C. 25 : 4
D. 35 : 2

Q. 145 Which school has the highest number of students scoring exactly 50% marks?

A. D
B. E
C. B
D. A

Q. 146 The total number of students scoring 50% or more marks is

A. 1250
B. 875
C. 775
D. 675

 

Study the following graph which shows income and expenditure of a company over the years 2005-2009 and answer question 147 to 150.

Q. 147 The difference in profit (₹ in crores) of the company during 2006 and 2007 is

A. 10
B. 15
C. 20
D. 25

Q. 148 In how many years was the income of the company less than the average income of the given years?

A. 4
B. 3
C. 2
D. 1

Q. 149 The percentage increase in expenditure to the company from 2007 to 2008 is

A. 20
B. 25
C. 30
D. 35

Q. 150 Profit of the company was maximum in the year

A. 2009
B. 2008
C. 2006
D. 2005

 

In question Nos. 151 to 155, some parts of the sentences have errors and some are correct. Find out which part of a sentence has an error and blacken the ovel [.] corresponding to the appropriate letter (A.B.C). If a sentence is free from error, blacken the ovel corresponding to (D) in the answer sheet.

Q. 151 Choose the correct option:

A. a
B. b
C. c
D. d

Q. 152 Choose the correct option:

A. a
B. b
C. c
D. d

Q. 153 Choose the correct option:

A. a
B. b
C. c
D. d

Q. 154 Choose the correct option:

A. a
B. b
C. c
D. d

Q. 155 Choose the correct option:

A. a
B. b
C. c
D. d

Directions: In Question Nos. 156 to 160, sentences are given with blanks to be field in with an appropriate word(s). Four alternatives are suggested for each question. Choose the correct alternative out of the four and indicate it by blackening the appropriate oval [•] in the Answer Sheet.

Q. 156 The student was punished for his _____.

A. impudence
B. prudence
C. modesty
D. elemency

Q. 157 My father was too _____ to push the heavy door.

A. faint
B. feeble
C. fragile
D. faltering

Q. 158 The flood damaged the books so much that it was impossible to _____ them.

A. retrieve
B. retrace
C. retract
D. retreat

Q. 159 His bungalow went through a make _____.

A. up
B. out
C. over
D. for

Q. 160 This auspicious beginning _____ well for a successful completion of our project.

A. attunes
B. argues
C. augurs
D. answers

Q. 161 choose the one which best expresses the meaning of the given word and mark it in the Answer Sheet
Persist

A. Resist
B. Leave
C. Quit
D. Insist

Q. 162 choose the one which best expresses the meaning of the given word and mark it in the Answer Sheet.
Eventually

A. previously
B. briefly
C. finally
D. successfully

Q. 163 choose the one which best expresses the meaning of the given word and mark it in the Answer Sheet
Impeccable

A. remarkable
B. unbelievable
C. flawless
D. displeasing

Q. 164 choose the word opposite in meaning to the given word and mark it in the Answer Sheet.
Predilection

A. Predicament
B. Afterthought
C. Aversion
D. Postponement

Q. 165 choose the word opposite in meaning to the given word and mark it in the Answer Sheet.
Pompous

A. Uppish
B. Humble
C. Meek
D. Grandiose

Q. 166 choose the word opposite in meaning to the given word and mark it in the Answer Sheet.
Serene

A. Calm
B. Angry
C. Ruffled
D. Bitter

Directions: In Question Nos. 167 to 171, four alternatives are given for the Idiom/Phrase underlined in the sentense. Choose the alternative which best expresses the meaning of the Idiom\Phrase and mark it in the Answer Sheet.

Q. 167 With great difficulty, he was able to carve out a niche for himself.

A. became a sculptor
B. did the best he could do
C. destroyed his career
D. developed a specific position for himself

Q. 168 You will succeed if you follow my advice to the letter.

A. about writing letters
B. written in the letter
C. in every detail
D. very thoughtfully

Q. 169 A critic’s work is to read between the lines.

A. to comprehend the meaning
B. to appreciate the inner beauty
C. to understand the inner meaning
D. to read carefully

Q. 170 Where discipline is concerned I put my foot down.

A. take a firm stand
B. take a light stand
C. take a heavy stand
D. take a shaky stand

Q. 171 The convict claimed innocence and stood his ground in spite of the repeated accusations.

A. knelt
B. surrendered
C. kept standing
D. refused to yield

Directions: In Question Nos. 172 to 181, a sentence/a part of the sentence is underlined. Below are given alternatives to the underlined part at (a), (b), (c) which may improve the sentence.  Choose the correct alternative. In case no improvement is needed your answer is (d). Mark your answer in the Answer Sheet.

Q. 172 Choose the correct option:

A. a
B. b
C. c
D. d

Q. 173 Choose the correct option:

A. a
B. b
C. c
D. d

Q. 174 Choose the correct option:

A. a
B. b
C. c
D. d

Q. 175 Choose the correct option:

A. a
B. b
C. c
D. d

Q. 176 Choose the correct option:

A. a
B. b
C. c
D. d

Q. 177 Choose the correct option:

A. a
B. b
C. c
D. d

Q. 178 Choose the correct option:

A. a
B. b
C. c
D. d

Q. 179 Choose the correct option:

A. a
B. b
C. c
D. d

Q. 180 Choose the correct option:

A. a
B. b
C. c
D. d

Q. 181 Choose the correct option:

A. a
B. b
C. c
D. d

Directions: In Question Nos. 182 to 188, out of the four alternatives, choose the one which can be substituted for the given words/sentences and indicate it by blackening the appropriate oval [•] in the Answer Sheet.

Q. 182 Belief in many gods

A. pantheism
B. monotheism
C. polytheism
D. atheism

Q. 183  A cluster of flowers on a branch

A. bouquet
B. inflorescence
C. wreath
D. incandescence

Q. 184  A person who believes that only selfishness motivates human actions

A. agnostic
B. cynic
C. sceptic
D. misogynist

Q. 185  A highly skilled musician

A. artiste
B. virtuoso
C. performer
D. diva

Q. 186  A method of boiling briefly to cook food slightly

A. steam
B. bake
C. saute
D. parboil

Q. 187  The group, especially in the arts, regarded as being the most experimental

A. avant-garde
B. iconoclast
C. revolutionary
D. nerd

Q. 188  One who helps people by giving them money or other aid

A. benefactor
B. beneficiary
C. tycoon
D. patriot

Directions: In Question Nos. 189 to 190, four words are given in each question, out of which only one word is correctly spelt. Find the correctly spelt word.

Q. 189 Choose the correct option:

A. Plebeian
B. Plibeian
C. Plebian
D. Plebiean

Q. 190 Choose the correct option:

A. Suroundings
B. Surroundings
C. Sarroundings
D. Surondings

PASSAGE – I (Q. NOS. 191-195)
As I stepped out of the train I felt unusually solitary since I was the only passenger to alight. I was accustomed to arriving in the summer, when holidaymakers throng coastal resorts and this was my first visit when the season was over. My destination was a little village which was eight miles by road. It took only a few minutes for me to come to the foot of the cliff path. When I reached the top I had left all signs of habitation behind me. I was surprised to notice that the sky was already a flame with the sunset. It seemed to be getting dark amazingly quickly. I was at a loss to account
for the exceptionally early end of daylight since I did not think I had walked unduly slowly. Then I recollected that on previous visits I had walked in high summer and how it was October. All at once it was night. The track was grassy and even in daylight showed up hardly at all. I was terrified of hurtling over the edge of the cliff to the rocks below. I felt my feet squelching and sticking in something soggy. Then I bumped into a little clump of trees that loomed up in front of me. I climbed up the nearest trunk and managed to find a tolerable comfortable fork to sit on. The waiting was spent by my attempts to identify the little stirrings and noises of animal life that I could hear. I grew colder and colder and managed to sleep only in uneasy fitful starts. At last when the moon came up I was on my way again.

Q. 191 The writer felt unusually solitary because

A. he was feeling very lonely without his family.
B. he was missing the company of other holiday-makers.
C. his destination was a little village eight miles away.
D. there was no one to meet him.

Q. 192 “I left all signs of habitation behind me.” This means that he

A. came to a place where there were very few houses.
B. was in front of a large collection of cottages
C. had come very far from places where people lived.
D. had just passed a remote village.

Q. 193 I became darker than the writer expected because

A. the nights are shorter in autumn than in summer.
B. the nights are longer in October than mid summer.
C. the train arrived later than usual.
D. he had walked unduly slowly.

Q. 194 The writer found it difficult to keep to the path because of

A. the darkness and narrowness of the path.
B. poor visibility and grassy track.
C. the darkness and his slow pace.
D. poor visibility and dew on grass

Q. 195 When he settled himself on the fork of the tree the writer ___________

A. had a sound sleep.
B. was disturbed by noises of animals.
C. was too afraid to sleep.
D. tried to sleep but without much success.

PASSAGE – II (Q. NOS. 196-200)
It is sad that in country after country, progress should become synonymous with an assault on nature. We who are a part of nature and dependent on her for every need, speak constantly about ‘exploiting’ nature. When the highest mountain in the world was climbed in 1953, Jawaharlal Nehru objected to the phrase conquest of Everest’ which he thought was arrogant. Is it surprising that this lack of consideration and the constant need to prove one’s superiority should be projected onto our treatment of our fellowmen? I remember Edward Thompson, a British writer and a good friend of India, once telling Mr. Gandhi that wildlife was fast disappearing. Remarked Mr. Gandhi: ‘It is decreasing in the jungles but it is increasing in the towns’. On the one hand the rich look askance at our continuing poverty; on the other they warn us against their own methods. We do not wish to impoverish the environment any further and yet we cannot forget the grim poverty of large numbers of people. Are not poverty and need the great polluters? For instance, unless we are in a position to provide employment and purchasing power for the daily necessities of the tribal people and those who live in and around our jungles, we cannot prevent them from combing the forest for food and livelihood, from poaching and from despoiling the vegetation.

Q. 196 At the beginning of the passage, the writer expresses her opinion that in many countries progress is synonymous with

A. development
B. utmost care for nature.
C. a balanced treatment of nature.
D. utmost cruelty to nature.

Q. 197 In the passage the term ‘exploiting’ nature suggests

A. regretfulness
B. sarcasm
C. destructive urge of man
D. greed of man

Q. 198 Nehru objected to the phrase ‘conquest of Everest’ since

A. it carries a war-like connotation.
B. it sounds pompous and boastful.
C. it depicts Everest as a victim.
D. Everest is unconquerable.

Q. 199 Gandhi’s statement ‘It is decreasing in the jungles but it is increasing in the towns.!’

A. Refers to wild animals’ decrease in the jungle
B. Refers to flora and fauna
C. Refers to man’s selfishness
D. Is a satirical comparison of man’s callousness to the animals

Q. 200 The writer is of opinion that tribal people can be prevented from combining forest for food

A. to provide employment
B. to increase purchasing power
C. by deterring them from poaching and despoiling vegetation
D. to provide employment and purchasing power for daily necessities 

 

 

 

Answer Sheet 
Question 1 2 3 4 5 6 7 8 9 10
Answer C C A B C B D B B B
Question 11 12 13 14 15 16 17 18 19 20
Answer D D D B B C C A B C
Question 21 22 23 24 25 26 27 28 29 30
Answer B C D D C C A D B A
Question 31 32 33 34 35 36 37 38 39 40
Answer C D A B B C C C C C
Question 41 42 43 44 45 46 47 48 49 50
Answer D C B A D B B D C D
Question 51 52 53 54 55 56 57 58 59 60
Answer B C C A B B A A A A
Question 61 62 63 64 65 66 67 68 69 70
Answer C D C A B C A A D C
Question 71 72 73 74 75 76 77 78 79 80
Answer A B A B A A C B D A
Question 81 82 83 84 85 86 87 88 89 90
Answer C B A D B B C A A C
Question 91 92 93 94 95 96 97 98 99 100
Answer C D C D D D D B D C
Question  101 102 103 104 105 106 107 108 109 110
Answer B B C C C C A B C A
Question  111 112 113 114 115 116 117 118 119 120
Answer B D D C C B A C C B
Question 121 122 123 124 125 126 127 128 129 130
Answer B B D A D D C D D C
Question 131 132 133 134 135 136 137 138 139 140
Answer A D C C A D D C C B
Question 141 142 143 144 145 146 147 148 149 150
Answer B A D B A C B C B C
Question 151 152 153 154 155 156 157 158 159 160
Answer A B B B B A B A C C
Question 161 162 163 164 165 166 167 168 169 170
Answer D D C C B C D C C A
Question 171 172 173 174 175 176 177 178 179 180
Answer D A D C A B A C B
Question 181 182 183 184 185 186 187 188 189 190
Answer C C B B B D A A A B
Question 191 192 193 194 195 196 197 198 199 200
Answer B C B D D D B B C D

 

 

×

Hello!

Click one of our representatives below to chat on WhatsApp or send us an email to info@vidhyarthidarpan.com

×